Multi-choice (From Website/Lectures & Tutorial) Flashcards

1
Q

What are foods high in potassium?

A
Foods high in POTASSIUM
#1: White Beans
#2: Dark Leafy Greens (Spinach)
#3: Baked Potatoes (With Skin)
#4: Dried Apricots
#5: Baked Acorn Squash
#6: Yogurt (Plain, Skim/Non-Fat)
#7: Fish (Salmon)
#8: Avocados
#9: Mushrooms (White)
#10: Bananas
How well did you know this?
1
Not at all
2
3
4
5
Perfectly
2
Q

What are foods high in MAGNESIUM?

A

Foods high in MAGNESIUM

#1: Dark Leafy Greens (Raw Spinach)
#2: Nuts and Seeds (Squash and Pumpkin Seeds)
#3: Fish (Mackerel)
#4: Beans and Lentils (Soy Beans)
#5: Whole Grains (Brown Rice
#6: Avocado
#7: Low-Fat Dairy (Plain Non Fat Yogurt)
#8: Bananas
#9: Dried Fruit (Figs)
#10: Dark Chocolate
How well did you know this?
1
Not at all
2
3
4
5
Perfectly
3
Q

What are foods high in IRON?

A

Foods High in Iron

#1: Mollusks (Clams, Mussels, Oysters)
#2: Liver (Pork, Chicken, Turkey, Lamb, Beef)
#3: Squash and Pumpkin Seeds
#4: Nuts (Cashew, Pine, Hazelnut, Peanut, Almond)
#5: Beef and Lamb (Lean Tenderloin)
#6: Beans and Pulses (White Beans, Lentils)
#7: Whole Grains, Fortified Cereals, and Bran
#8: Dark Leafy Greens (Spinach, Swiss Chard)
#9: Dark Chocolate and Cocoa Powder
#10: Tofu
How well did you know this?
1
Not at all
2
3
4
5
Perfectly
4
Q

During her first clinic visit, the patient states that she is divorced and remarried to a divorced man with two children. The two stepchildren alternate living with their father and their natural mother. The nurse recognizes that the patient is describing the family model called:

A

Rationale: The binuclear family is a postdivorced family in which the children are members of two nuclear households and alternate living between the two homes. The extended family consists of a couple who shares household responsibilities, chores, and expenses with parents, siblings, or other relatives living in the same home. The extended kin network family consists of two families living in close proximity. The family shares a social support network and resources. The stepparent family includes a biological parent, with children, and a new spouse, all living together.

How well did you know this?
1
Not at all
2
3
4
5
Perfectly
5
Q

A nurse is working in a prenatal clinic where many of the patients come from a variety of cultural backgrounds. Which of the following would assist the nurse in understanding the beliefs, values, and customs of these patients?

A

Rationale: Using a cultural assessment tool will assist the nurse in gathering information about health practices that are based on the patient’s beliefs, values, and customs. A family assessment is a collection of data about the family’s type, structure, level of functioning, support system, and needs. Taboos focus only on behaviors that the patient should avoid. An herbalist may be seen as a healer within the Asian culture

How well did you know this?
1
Not at all
2
3
4
5
Perfectly
6
Q

An Asian couple is excited about the upcoming birth of their baby. The nurse conducts a cultural assessment and understands that since their arrival in the United States, they have adapted to a new cultural norm in preparation for the birth of their fist baby. This process is called:

A

Rationale: Acculturation is the process by which people adapt to a new cultural norm. Cultural competency is acquiring the skills and knowledge necessary to respect and work with individuals from different cultures. Ethnocentrism is the conviction that the values and beliefs of one’s own cultural group are the best or only acceptable ones. Enculturation occurs when cultural norms and beliefs are passed down from one generation to the next.

How well did you know this?
1
Not at all
2
3
4
5
Perfectly
7
Q

When the nurse is assessing the patient’s use of complementary therapies, the nurse will be most concerned with the patient who uses: (Select all that apply.)

A

Rationale: Acupuncture is a complementary therapy, and often is used with conventional medical care. Herbal therapies, homeopathy, and magnet therapy are considered alternative therapy, and have not undergone rigorous scientific testing in this country.

How well did you know this?
1
Not at all
2
3
4
5
Perfectly
8
Q

During her first clinic visit, the patient states that she wants to use alternative therapies during her pregnancy. The nurse knows that one resource for evaluating alternative therapies is the:

A

Rationale: The National Center for Complementary and Alternative Medicine (NCCAM) promotes research into complementary and alternative therapies, and dissemination of information to consumers.

How well did you know this?
1
Not at all
2
3
4
5
Perfectly
9
Q

During a prenatal home visit, the patient informs the nurse that she cannot come to the evening childbirth preparation classes because the night air may cause her to become ill. The nurse recognizes that the patient’s statement reflects a cultural:

A

Rationale: Taboos refer to behaviors to be avoided. The equilibrium model of health is based on a concept of life balances such as “hot” and “cold.” Health practices are behaviors that are influenced by many factors, including home remedies and folk beliefs. Attitudes about pregnancy dictate whether the pregnancy is viewed as a normal state of wellness or a state of increased vulnerability and illness.

How well did you know this?
1
Not at all
2
3
4
5
Perfectly
10
Q

The nurse is working with a patient to identify a complementary therapy that may help control pregnancy-induced hypertension. The patient asks, “Is there a system where I can learn to focus and control my blood pressure with my thoughts?” The nurse recommends:

A

Rationale: Biofeedback helps patients to control their physiologic responses based on the concept that the mind controls the body. Naturopathy is a healing system that uses natural means to prevent and treat diseases such as nutrition, hydrotherapy, and homeopathy. Hypnosis is a state of mental and physical relaxation during which a patient is very open to suggestions. Chiropractic care is based on the concepts of manipulation to address health problems thought to be a result of abnormal nerve transmission in the spine.

How well did you know this?
1
Not at all
2
3
4
5
Perfectly
11
Q

An adolescent boy asks, “Does the scrotum have a function?” The nurse’s best response is:

A

Rationale: The scrotum’s main purpose is to protect the testes and maintain a temperature that is lower than body temperature so spermatogenesis can occur. Because it is sensitive to touch, pain, and pressure, the scrotum defends against potential harm to the testes. Ejaculation occurs with sexual stimulation and expulsion of semen by rhythmic contractions of the penile muscles.

How well did you know this?
1
Not at all
2
3
4
5
Perfectly
12
Q

A nurse is teaching a sex education class about the female reproductive system for sixth-grade girls in a local middle school. The nurse explains that the primary components of the external female reproductive system are:

A

Rationale: Mons, labia, and the clitoris make up part of the external female reproductive system. They can be seen directly and inspected. The vaginal canal, urethra, and vagina are all part of the internal female reproductive system.

How well did you know this?
1
Not at all
2
3
4
5
Perfectly
13
Q

A nurse is teaching a class about the reproductive system, and correctly states that the purpose of the labia minora is to:

A

Rationale: The labia minora contain sebaceous glands that lubricate the vulvar skin and have a bactericidal effect. The clitoris assists with sexual stimulation by secreting smegma. Skene’s glands aid in lubricating the vaginal opening. Bartholin’s glands secrete a mucus that aids in the viability and motility of the sperm.

How well did you know this?
1
Not at all
2
3
4
5
Perfectly
14
Q

The nurse is assessing the pH level of the vaginal environment of a 26-year-old patient. Which of the following would be an expected finding for this patient?

A

Rationale: The normal range for vaginal pH levels for a patient during the childbearing years is 4.0–5.0. The environment of the vagina is acidic. Below 4.0 is too low and above 5.0 is too high.

How well did you know this?
1
Not at all
2
3
4
5
Perfectly
15
Q

A nurse is teaching a class for adolescents about the female reproductive system. When the nurse asks the class what the function of the vagina is, she knows that further teaching is necessary when a student answers:

A

Rationale: The vagina does not protect the labia minora. The vagina is an internal structure, and the labia minora are an external structure. The vagina provides a passageway for menstrual flow, sperm, and the fetus, and helps protect against infection.

How well did you know this?
1
Not at all
2
3
4
5
Perfectly
16
Q

A male patient is having some problems with infertility, and is waiting for a report on his semen sample. The nurse knows that the problem may not be related to the pH level of the semen based on the following pH lab value:

A

Rationale: Adequate pH level for semen is 7.5, which aids in the effective transport of sperm. A pH of 4.5–6.5 is too low.

How well did you know this?
1
Not at all
2
3
4
5
Perfectly
17
Q

A patient relates that she cracked her pelvis in a car accident and wants to know the purpose of the pelvis during childbirth. The nurse’s best response is:

A

Rationale: The two main purposes of the bony pelvis are to support and protect the contents of the pelvic cavity and to form a relatively fixed axis for the birth passage. The muscles of the pelvic floor help overcome the force of gravity and provide stability. The ovum is fertilized in the fallopian tubes. The ovaries are the main site for estrogen and progesterone secretion.

How well did you know this?
1
Not at all
2
3
4
5
Perfectly
18
Q

A nurse is reviewing a patient’s record and notices a physician’s report of a malformation in one of the fallopian tubes. The patient is most at risk for:

A

Rationale: A malformation within the fallopian tube may decrease the ability for the ovum to pass through, resulting in infertility, ectopic pregnancy, or even sterility. Urinary tract infections and early menopause are not related to fallopian tube malformations. Estrogen is secreted by the ovaries.

How well did you know this?
1
Not at all
2
3
4
5
Perfectly
19
Q

Which statement best describes the correct order of the four phases of the menstrual cycle?

A

Rationale: Menstrual, proliferative, secretory, ischemic is the correct order for the four phases of the menstrual cycle. There is no luteal or follicular phase in the menstrual cycle.

How well did you know this?
1
Not at all
2
3
4
5
Perfectly
20
Q

A nurse is teaching a couple about the process of fertilization. Which statement by the couple would indicate understanding of fertilization?

“Fertilization takes place in the ampulla (outer third) of the fallopian tube.”

A

Rationale: The ampulla is the outer third of the fallopian tube. The word comes from the Latin word ampullae, meaning “jug.” Ampulla is a general term used in anatomy to designate a flasklike dilation of a tubular structure. The fimbria is a funnel-like enlargement with many fingerlike projections reaching out to the ovary at the end of the fallopian tube. The isthmus is the straight and narrow portion of the inner third of the fallopian tube, and it opens into the uterus; this is the site for tubal ligation.

How well did you know this?
1
Not at all
2
3
4
5
Perfectly
21
Q

A pregnant patient is concerned about a blow to the abdomen if she continues to play basketball during her pregnancy. The nurse’s response is based upon her knowledge of which of the following facts concerning amniotic fluid?

A

Amniotic fluid functions as a cushion to protect against mechanical injury.

Rationale: During pregnancy, the amniotic fluid protects against injury. After 20 weeks of pregnancy, fluid volume ranges from 700 to 1000 mL. Some of the amniotic fluid is contributed by the fetus’s excreting urine. Amniotic fluid is slightly alkaline.

How well did you know this?
1
Not at all
2
3
4
5
Perfectly
22
Q

A patient states that she had a spontaneous abortion 12 months ago and asks if her hormones may have contributed to the loss of the pregnancy. The nurse’s response is based upon which fact?

A

Progesterone decreases the contractility of the uterus.

Rationale: Progesterone decreases the contractility of the uterus, thus preventing uterine contractions that might cause spontaneous abortion. Progesterone must be present in high levels for implantation to occur. After 10 weeks, the placenta takes over the production of progesterone. hCG reaches its maximum level at 50–70 days gestation.

How well did you know this?
1
Not at all
2
3
4
5
Perfectly
23
Q

A nurse is teaching a group of student nurses about amniotic fluid. Which statement by the student nurse reflects an understanding of the fetus’s contribution to the quantity of amniotic fluid?

A

Correct Answer:
“The fetus contributes to the volume of amniotic fluid by excreting urine.”
Rationale: “The fetus contributes to the volume of amniotic fluid by excreting urine.” Approximately 400 mL of amniotic fluid flows out of the fetal lungs each day. The fetus swallows about 600 mL of the fluid in 24 hours. A normal volume of amniotic fluid is necessary for good fetal movement. Normal movement is necessary for good musculoskeletal development.

How well did you know this?
1
Not at all
2
3
4
5
Perfectly
24
Q

The nurse is preparing an educational workshop on fetal development. Which statement by the student would require the nurse to explain further?

A

Correct Answer:
“The umbilical cord is made of two veins and one artery.”
Rationale: A nuchal cord exists when the umbilical cord encircles the fetal neck. Fetal movement causes the umbilical cord to knot. The umbilical cord is made of two arteries and one vein. The high blood volume and Wharton’s jelly content of the umbilical cord prevents compression of the cord.

How well did you know this?
1
Not at all
2
3
4
5
Perfectly
25
Q

At 17 weeks pregnant, a mother asks the nurse questions about the development of her baby. The mother states that it may be too early to visualize any body structures via ultrasound. The nurse’s best response in relation to fetal development at 17 weeks is:

A

Your Answer:
The earlobes are soft with little cartilage.

Rationale: Differentiation of hard and soft palate are structures developed by 17 weeks gestation. Myelination of the spinal cord begins at 20 weeks gestation. Soft earlobes with little cartilage develop at 36 weeks gestation. Teeth form hard tissue (enamel) at 18 weeks gestation.

How well did you know this?
1
Not at all
2
3
4
5
Perfectly
26
Q

The nurse is preparing an educational workshop on the time frames for fetal exposure to potential teratogens. Which defect is most likely to occur at seven weeks gestation?

A

Correct Answer:
A cleft palate
Rationale: Cleft palate is the defect most likely to occur at seven weeks gestation. Shortening of fingers and toes most likely occurs at eight weeks gestation. Cleft lip most likely occurs at six weeks gestation. Septal or aortic abnormalities are most likely to occur at six weeks gestation.

How well did you know this?
1
Not at all
2
3
4
5
Perfectly
27
Q

A patient in the prenatal clinic tells the nurse that her sister has twins and is concerned that she may also have twins. The nurse’s response is based upon which fact?

A

Your Answer:
Genetic factors in the mother may lead to elevated serum gonadotropin levels, causing double ovulation.
Rationale: A genetic factor that results in elevated serum gonadotropin levels may cause double ovulation. The chance of dizygotic twins increases with maternal age up to about 35 years and then decreases abruptly. Monozygotic twins share a common amniotic sac only if division occurs 7–13 days after fertilization. The survival rate of dizygotic twins is about 10% lower than that of monozygotic twins.

How well did you know this?
1
Not at all
2
3
4
5
Perfectly
28
Q

A nurse is instructing her students where to listen for a uterine souffle. Each student has placed the fetoscope on the pregnant women’s abdomen. Based on the following placements of the fetoscopes, the student most likely to hear a uterine souffle is the one who placed her fetoscope on which of the following areas?

A

Your Answer:

Just above the symphysis pubis

How well did you know this?
1
Not at all
2
3
4
5
Perfectly
29
Q

The nurse must choose an appropriate location and method of taking a sexual health history. Which of the following would be the best choice?

A

Correct Answer:
An interview in the examination room with the door closed
Rationale: A quiet, private environment is important for conducting a sexual history. Waiting areas and interviews in the presence of other individuals are not appropriate. The patient’s comfort is important. The nurse uses effective communication skills to elicit this important data.

How well did you know this?
1
Not at all
2
3
4
5
Perfectly
30
Q

Which statement made by a preteen girl indicates successful adaptation to menarche?

A

Correct Answer:
“My cycle should occur every 28 days and last about five days.”
Rationale: Young girls will adapt successfully to the initiation of menses and cope better when they have correct information concerning cycle length, amount of flow, length of menses, and management issues.

How well did you know this?
1
Not at all
2
3
4
5
Perfectly
31
Q

A 20-year-old woman reports to the nurse about her menstrual flow. Which statement made by the patient would be cause for further investigation?

A

Correct Answer:
“I had to change my maxi pad at least 14 times today.”
Rationale: Saturation of a maxi pad more often than every one to two hours, especially for more than a day, could put a patient at risk for anemia.

How well did you know this?
1
Not at all
2
3
4
5
Perfectly
32
Q

Which content area is important to include in a teaching plan presented to young girls to assist them in preventing toxic shock syndrome?

A

Correct Answer:
Change tampons every three to six hours and wash hands before insertion.
Rationale: Toxic shock syndrome is associated with the use of super-absorbent tampons and the bacteria Staphylococcus aureus. This bacteria is commonly found on the hands. Handwashing may avoid the transmission to the genital tract.

How well did you know this?
1
Not at all
2
3
4
5
Perfectly
33
Q

When asked if douching is appropriate, how should the nurse respond?

A

Correct Answer:
“Douching is associated with susceptibility to infection.”
Rationale: Douching washes away the natural cervical mucus plug and changes the vaginal flora, increasing susceptibility to infection. It should be avoided during the menstrual cycle, as the cervix is dilated and fluid may be propelled into the uterus.

How well did you know this?
1
Not at all
2
3
4
5
Perfectly
34
Q

A young woman states that her menstrual cycle occurs every 20 days. What additional data should the nurse gather?

A

Correct Answer:
The amount of flow per cycle
Rationale: Polymenorrhea is characterized by bleeding occurring at intervals of less than 20 days. It is important to ascertain the amount of flow per cycle to determine if menorrhagia is present. Type of protection, number of sexual partners, and mother’s age at menopause are not applicable to this scenario.

How well did you know this?
1
Not at all
2
3
4
5
Perfectly
35
Q

A 16-year-old patient states that she has never started her menstrual cycle. The nurse asks about marked weight loss, excessive exercise, and prolonged stress. Which cause of amenorrhea is he inquiring about?

Your Answer:
Hypothalmic dysfunction

A

Your Answer:
Hypothalmic dysfunction
Rationale: Hypothalmic dysfunction is characterized by marked weight loss, excessive exercise, and prolonged stress. Pituitary dysfunction usually is related to medication used to treat anxiety or psychiatric disorders, head trauma, and cancer. Ovarian failure can be related to Turner’s syndrome, exposure to radiation, chemotherapy, viral infection, and surgical removal of the ovary. Anatomic abnormalities that would cause amenorrhea would be disorders such as congenital absence of the uterus, ovaries, or vagina.

How well did you know this?
1
Not at all
2
3
4
5
Perfectly
36
Q

A patient at the family planning clinic is diagnosed with primary dysmenorrhea. What should be included in the nurse’s teaching plan for nonpharmacologic comfort measures?

A

Correct Answer:
Balanced meals and adequate rest
Rationale: Self-care measures to treat dysmenorrhea would include regular exercise, rest, application of heat, and good nutrition.

How well did you know this?
1
Not at all
2
3
4
5
Perfectly
37
Q

The nurse is taking a neurologic health history from a female patient who is concerned with symptoms of premenstrual syndrome. Which symptom should the nurse inquire?

A

Correct Answer:
Vertigo
Rationale: Symptoms of PMS occur between ovulation and the onset of menses, and can affect all body systems. Neurologic symptoms include migraine headache, vertigo, and syncope.

How well did you know this?
1
Not at all
2
3
4
5
Perfectly
38
Q

A married couple is planning a pregnancy. She suffers from premenstrual syndrome, and asks the nurse which relief measure would be appropriate. What is the nurse’s best response?

A

Correct Answers:
Increase intake of complex carbohydrates.
Exercise.
Rationale: Women considering pregnancy should refrain from pharmacologic treatment of PMS. Methylxanthine-containing foods should be decreased, and intake of complex carbohydrates should be increased. Exercise has been shown to increase the body’s natural endorphins to relieve pain, and boosts energy and mood as well.

How well did you know this?
1
Not at all
2
3
4
5
Perfectly
39
Q

A nurse is teaching a group of college women about prevention of toxic shock syndrome (TSS). Which of the following statements should be included in the seminar?

A

Correct Answer:
Women with a history of TSS should never use tampons.
Rationale:
- Usually, the causative organism of TSS is is a toxin released by a strain of Pseudomonas aeruginosa. (False - The usual causative organism is a toxin released by a strain of Staphylococcus aureus)
- Hypertension is often seen in patients with TSS (False - TSS is associated with hypotension)
- To avoid TSS, tampons should be changed every 6 to 9 hours (False - tampons should be changed every 3 - 6 hours to avoid TSS)
- Women with a history of TSS should never use tampons. (True)

How well did you know this?
1
Not at all
2
3
4
5
Perfectly
40
Q

A nurse is educating a group of patients about the risk factors for developing toxic shock syndrome (TSS). Which statement by a patient would indicate that she would benefit from more teaching on the risk factors of TSS?

A

Correct Answer:
“I should use super-absorbent tampons only at night during my period.”
Rationale: “I should use super-absorbent tampons only at night during my period” indicates that a patient would benefit from more teaching on the risk factors of TSS. The risk of TSS is reduced when alternating between pads and tampons. The risk of TSS is reduced when a diaphragm or cervical cap is not used during the menses. The risk of TSS is reduced when tampons are changed every three to six hours during the menses.

How well did you know this?
1
Not at all
2
3
4
5
Perfectly
41
Q

A patient describes breast swelling and tenderness. Gathering what piece of data would be most important?

A

Correct Answer:
Timing of the symptoms
Rationale: The breast undergoes regular cyclical changes in response to hormonal stimulation. The nurse will want to determine when the swelling and tenderness occurs within the menstrual cycle. Birth control method, method of BSE, and diet history may contribute to the database, but do not have priority.

How well did you know this?
1
Not at all
2
3
4
5
Perfectly
42
Q

A 24-year-old female patient is being treated with tinidazole for bacterial vaginosis. Which information should the nurse include in this patient’s discharge instructions?

A

Correct Answer:
Alcohol should be avoided when taking tinidazole
Rationale:
- Alcohol should be avoided when taking tinidazole (true)
- Tinidazole may be administered orally in tablet form or vaginally as a cream (false - this medication is administered orally)
- Side effects of tinidazole include white or gray vaginal discharge with a foul odor described as “fishy.” (false - these are signs and symptoms of bacterial vaginosis infection)
- Tinidazole is contraindicated in pregnant patients (false)

How well did you know this?
1
Not at all
2
3
4
5
Perfectly
43
Q

In order to assess the origin of galactorrhea, the nurse must gather data about what?

A

Correct Answer:
Color and consistency of discharge
Rationale: Galactorrhea is nipple discharge not associated with lactation. Determining color and consistency of that discharge assists the nurse in distinguishing between physiologic discharge and pathologic discharge. Masses, dimpling, and presence of rash will not give the nurse data concerning galactorrhea.

How well did you know this?
1
Not at all
2
3
4
5
Perfectly
44
Q

A 32-year-old woman who is at 29 weeks’ gestation presents to the clinic for a routine prenatal visit. Her physician orders a urinalysis, which reveals asymptomatic bacteriuria (ASB). Which of the following plans of care should the nurse anticipate?

A

Correct Answer:
Explaining that treatment will be necessary even if fever, chills, or any other signs and symptoms of infection do not develop
Rationale:
- Instructing the patient about the risks of developing a descending urinary tract infection (false - urinary tract infections are ascending)
- Explaining that treatment will be necessary even if fever, chills, or any other signs and symptoms of infection do not develop (true - because untreated ASB can lead to pyelonephritis in the pregnant woman and low birth weight in the newborn, treatment will be necessary despite the absence of symptoms)
- Preparing for administration of a broad-spectrum antibiotic, as ASB is usually caused by two separate organisms (false - ASB is almost always caused by a single organism, typically Escherichia coli, and treatment should target the causative organism)
- Facilitating additional diagnostic testing, including a blood culture, and encouraging strict bedrest until antibiotic therapy is completed (false - in the treatment of pyelonephritis, a blood culture is necessary and bed rest is advised)

How well did you know this?
1
Not at all
2
3
4
5
Perfectly
45
Q

The nurse is teaching a group of female teenagers about prevention of sexually transmitted infection (STI). Which student’s statement suggests the need for additional teaching?

A

Your Answer:
“If I test positive for the human papillomavirus (HPV), my sexual partner and I both have to be treated with medication”
Rationale:
- “I can get venereal warts by having anal sex” (false - this statement is correct) - “The same STI that causes genital warts can cause cancer of the cervix” (false - this statement is correct)
- “If I test positive for the human papillomavirus (HPV), my sexual partner and I both have to be treated with medication” (true - this is incorrect; sex partners do not require treatment unless large lesions are present)
- “Human papillomavirus (HPV) can be spread from one person to another during oral sex” (false - this statement is correct)

How well did you know this?
1
Not at all
2
3
4
5
Perfectly
46
Q

Upon reviewing the patient’s chart, which piece of data would suggest to the nurse the doctor may diagnose endometriosis?

A

Correct Answer:
Pelvic cramping and dyspareunia
Rationale: The most common symptom of endometriosis is pelvic pain. Other symptoms include dysmenorrhea, dyspareunia, abnormal uterine bleeding, and infertility.

How well did you know this?
1
Not at all
2
3
4
5
Perfectly
47
Q

The community health nurse is creating an educational brochure about sexually transmitted infections (STIs). Which information should the nurse include when discussing the two types of herpes infections?

A

Correct Answer:
Both HSV-1 and HSV-2 can cause genital herpes
Rationale:
- Both HSV-1 and HSV-2 can cause genital herpes (true)
- HSV-1 may cause cold sores but does not cause genital herpes (false - HSV-1 can lead to genital herpes through oral–genital contact)
- If administered early in the course of treatment, acyclovir, valacyclovir, or famciclovir are effective cures for either type of HSV (false - no known cure for herpes exists)
- Emotional stress does not impact the recurrence of HSV (false)

How well did you know this?
1
Not at all
2
3
4
5
Perfectly
48
Q

A married woman presents to the clinic with complaints of grayish vaginal discharge with a “fishy” odor. Which of the following statements demonstrates that your teaching regarding BV has been understood?

A

Correct Answer:
“It is not necessary to treat my partner.”
Rationale: Bacterial vaginosis is not a sexually transmitted disease. Studies have not found a lower risk of reinfection with treating male partners. Metrogel (Flagyl) now is considered safe in pregnancy. However, alcohol should be avoided while on this medication, due to ill side effects from the combination.

How well did you know this?
1
Not at all
2
3
4
5
Perfectly
49
Q

The nurse is teaching a class on infertility. Which statement correctly describes infertility?

A

Your Answer:
Lack of conception despite unprotected sexual intercourse for at least 12 months
Rationale: Infertility is defined as lack of conception despite unprotected sexual intercourse for at least 12 months. When there is an absolute factor preventing reproduction, it is described as sterility. A couple having difficulty conceiving because both partners have decreased fertility is subfertility (reduced ability to conceive). Being a woman over 35 and having difficulty conceiving is a possible risk factor for infertility but does not describe infertility.

How well did you know this?
1
Not at all
2
3
4
5
Perfectly
50
Q

The nurse is teaching a couple about fertility during the female reproductive cycle. The couple asks the nurse, “When is the most fertile time for intercourse?” What is the best response by the nurse?

A

Your Answer:
Three days before and after ovulation
Rationale: The best response by the nurse is, “Three days before and after ovulation.” Follicular phase is the first fourteen days of the ovarian cycle, and includes both fertile and nonfertile days. Twelve to 24 hours before and after ovulation is too short of a time frame. The luteal phase is the last 14 days of the ovarian cycle, and includes both fertile and nonfertile days.

How well did you know this?
1
Not at all
2
3
4
5
Perfectly
51
Q

A nurse is reviewing a basal body temperature chart. Which change would indicate probable ovulation?

A

Your Answer:
Decrease in temperature followed by an increase for several days
Rationale: A decrease in temperature is followed by an increase in temperature occurring about a day after ovulation, which is maintained for several days.

How well did you know this?
1
Not at all
2
3
4
5
Perfectly
52
Q

A nurse is teaching a patient with infertility about the medication Clomid (clomiphene citrate). What side effect of Clomid would indicate the need to discontinue the medication?

A

Correct Answer:
Visual disturbances
Rationale: Visual disturbances are a contraindication for continuing the medication. Hot flashes are related to the antiestrogenic effects of Clomid, and do not merit discontinuation. Abdominal distention and headaches also are anticipated side effects.

How well did you know this?
1
Not at all
2
3
4
5
Perfectly
53
Q

A 34-year-old female patient has been diagnosed with recurrent pregnancy loss (RPL). Which of the following statements accurately represents this condition?

A

Your Answer:
RPL is another term for infertility
Rationale:
- RPL is another term for infertility (false)
- Research has demonstrated that there is no link between RPL and autoimmune disorders (false)
- RPL is defined by three or more failed pregnancies (false)
- Thrombotic causes are associated with RPL (true)

How well did you know this?
1
Not at all
2
3
4
5
Perfectly
54
Q

A nurse is teaching a class on the environmental factors affecting male fertility. Which factors could reduce the sperm count? (Select all that apply.)

A

Correct Answers:
Heavy use of marijuana, alcohol, or cocaine

Exposure to pesticides
Correct.

Rationale: Exposure to pesticides could reduce the sperm count, as could heavy use of marijuana, alcohol, or cocaine. All have been shown to depress sperm count and testosterone levels. Use of hot tubs and wearing of briefs are not proven factors in reducing sperm count, although they are thought to decrease fertility. Cigarette smoking decreases sperm motility.

How well did you know this?
1
Not at all
2
3
4
5
Perfectly
55
Q

A nurse is teaching an educational seminar about genetic disorders. The nurse correctly responds to a question about mosaicism by stating that individuals who have this chromosomal abnormality also may have which characteristic?

A

Correct Answer:
Normal intelligence
Rationale: Severe mental retardation, congenital heart defects, and underdeveloped sex characteristics describe trisomic individuals.

How well did you know this?
1
Not at all
2
3
4
5
Perfectly
56
Q

Which disorder would not result in a higher-than-normal AFP level?

A

Correct Answer:
Down syndrome
Rationale: Open neural tube defects, anencephaly, omphalocele, gastroschisis, and multiple gestations cause higher-than-normal AFP levels. Down syndrome causes lower-than-expected levels.

How well did you know this?
1
Not at all
2
3
4
5
Perfectly
57
Q

Which test or workup do you think is most appropriate for a patient who has no children, a history of two ectopic pregnancies, and currently reports an inability to get pregnant?

A

Correct Answer:
Hysterosalpingography
Rationale: Ectopic pregnancies can cause scarring in the fallopian tubes, thereby causing an obstruction between the ovaries and the uterus. Hysterosalpingography is the test to determine tubal patency and uterine structure.

How well did you know this?
1
Not at all
2
3
4
5
Perfectly
58
Q

A nurse is working with four patients interested in in vitro fertilization. Which patient would be a good candidate for using a gestational carrier?

A

Rationale: IVF using a gestational carrier is appropriate for the woman who is genetically sound but unable to carry a pregnancy due to absence of her uterus. The patient with tubal blockage is a candidate for in vitro fertilization with intrauterine embryo transfer. The patient whose husband has a very low sperm count is a candidate for in vitro fertilization. The patient with isoimmunization is a high-risk pregnancy, but this condition does not preclude pregnancy.

Assessment; Health Promotion and Maintenance; Analysis

How well did you know this?
1
Not at all
2
3
4
5
Perfectly
59
Q

Which patient would be most likely to experience a Bradley type labor and birth?

A

Correct Answer:
23 Y/O G1P0 – Married, supportive partner
Rationale: The Bradley method’s foundation is husband-coached childbirth. Lamaze is a disassociation type with incorporation of breathing patterns; Hypnobirth incorporates hypnosis type state; Kitzinger uses chest breathing and relaxation.

How well did you know this?
1
Not at all
2
3
4
5
Perfectly
60
Q

Which statement indicates the need for further teaching as it relates to vitamin recommendations in pregnancy?

A

Correct Answers:
“The doctor prescribed vitamins for me, but they make me sick so I quit taking them.”

“My doctor said that I need to take vitamins to stay healthy. I am taking the ones he prescribed, as well as my own multivitamin.”
Rationale: Increased amounts of certain vitamins and nutrients are recommended in pregnancy. However, there is a possibility of an overdose, which could cause fetal problems. It is important to determine what amount is needed and to keep track of intake.

How well did you know this?
1
Not at all
2
3
4
5
Perfectly
61
Q

Which statement indicates the need for further teaching regarding pregnancy and exercise? (Select all that apply.)

A

Correct Answers:
“I have gained so much weight during my pregnancy. I am going to start jogging.”

“I feel so tired after my workouts; I need to push myself to remain in shape.”
Correct.

Rationale: A regular exercise plan should be initiated and in place three months prior to pregnancy. Initiating or increasing exercise during pregnancy is not generally recommended.

How well did you know this?
1
Not at all
2
3
4
5
Perfectly
62
Q

Regarding cesarean section and vaginal birth after cesarean (VBAC), which statement is accurate?

A

Your Answer:
According to the American College of Obstetricians and Gynecologists (ACOG), VBAC is a safe alternative for most women
Rationale:
- According to the American College of Obstetricians and Gynecologists (ACOG), VBAC is a safe alternative for most women (true)
- The Agency on Healthcare Research and Quality published findings that 95% of VBACs were successful (false)
- Women over the age of 40 give birth via cesarean at rates of 80% (false)
- VBAC should be considered only in women under the age of 35 (false)

How well did you know this?
1
Not at all
2
3
4
5
Perfectly
63
Q

A married couple are apprehensive about their labor and delivery. They report a loss of control and a reckless regard to their wishes with their previous hospital birth. Which nursing interventions/responses would be most appropriate? (Select all that apply.)

A

Correct Answers:
Assisting this couple with a birth plan.

Offering information to allow for informed choices.
Correct.

Rationale: A birth plan allows for the parents’ wishes to be made known to those providing care. It is important for the nurse to provide means for a couple to make plans and gain comfort about their care. It is good to suggest that they discuss their wishes with their physician; however, it is within nursing’s scope to further help them. They are not interested in routine care; they are concerned with loss of control and not having their wishes respected. Patient teaching is important. Allowing the couple to make decisions is their wish. It is important that they are making decisions with appropriate and true information.

How well did you know this?
1
Not at all
2
3
4
5
Perfectly
64
Q

A married couple have decided to have their older children present in the delivery room. During labor, the patient is verbalizing and writhing in pain. There is bloody show, and her youngest child begins to cry and say, “No! No!” What would be an appropriate nursing intervention? (Select all that apply.)

A

Correct Answers:
Encourage the support person to take the child from the room.

Avoid generalized reassurance.
Correct.

Rationale: Children should have their own support person present. They should never be made to feel uncomfortable. They should have the option to come and go from the room as they desire.

How well did you know this?
1
Not at all
2
3
4
5
Perfectly
65
Q

A co-worker is teaching prenatal classes. She is covering physiologic changes of pregnancy. Select the changes that are correctly matched with the appropriate times to present them. (Select all that apply.)

A

Correct Answers:
Enlarged, tender breasts; first trimester

Vasodilation and its association with hemorrhoids, etc.; first trimester
Rationale: Enlarged, tender breasts occur in the first trimester. Quickening occurs prior to the third trimester. This is an appropriate time to address vasodilation and its association with hemorrhoids, etc. Uterine involution occurs after delivery. Better to wait until third trimester for teaching on this topic.

How well did you know this?
1
Not at all
2
3
4
5
Perfectly
66
Q

Which person would be the best candidate to provide support to a sibling attending a birth?

A

Correct Answer:
The child’s close aunt, who has had children before.
Rationale: Siblings should have their own separate support person with whom they are comfortable, who is familiar with the birthing process and is comfortable talking about sexuality.

How well did you know this?
1
Not at all
2
3
4
5
Perfectly
67
Q

A married woman has given birth to a seven-pound, seven-ounce baby boy. She is breast-feeding. You enter the room to provide care. Her husband states, “I don’t know why she has to do that. None of our other kids like me because they all were breastfed. They want nothing to do with me.” What nursing intervention would be most appropriate? (Select all that apply.)

A

Correct Answers:
Encourage the husband to rock and hold the baby.

Encourage the husband to provide other baby care, such as diapering, burping, changing, etc.
Correct.

Rationale: Encourage activities that promote the father’s involvement and bonding. Informing him that breastfeeding is best and that he needs to support his wife in her decision doesn’t recognize his needs. They have decided to breast-feed, so offering formula is not an option. Exclusive breastfeeding is best in the first few weeks. There are better ways to incorporate the husband in care.

How well did you know this?
1
Not at all
2
3
4
5
Perfectly
68
Q

A married woman is having a scheduled repeat cesarean delivery. She states that she doesn’t know if she can do that again. Upon further questioning, she states that she “labored for 29 hours then had an emergency C-section.” She wasn’t able to see her baby until hours later, and had terrible pain in the recovery period. She states that she was unable to provide infant care for many days due to her discomfort. Choose an appropriate nursing intervention. (Select all that apply.)

A

Correct Answers:
Teach the patient about C-sections and plan of care.

Inform the patient that every delivery is different. She will not be so fatigued going into things this time, and that should help with the pain and recovery time.
Rationale: Don’t provide a false sense of security or over-generalized reassurance. This doesn’t address her concerns. Subsequent C-sections often are less painful than the first. Fatigue and fear play a part in pain and pain perception. There previously may not have been time for teaching. It is good to reiterate and teach to help eliminate some fear. Although the same incision line may be used, there still may be discomfort.

How well did you know this?
1
Not at all
2
3
4
5
Perfectly
69
Q

A 25-year-old patient at 18 weeks gestation has returned to the clinic for her second prenatal visit. Her initial pulse was 60. The nurse can expect her pulse to be bpm at term.

A

Correct Answer:
70-75
Rationale: The pulse may increase by 10–15 bpm at term.

How well did you know this?
1
Not at all
2
3
4
5
Perfectly
70
Q

A nurse is teaching a prenatal patient about cardiovascular changes during pregnancy. The patient asks the nurse why she becomes dizzy when getting out of a chair or out of bed. What is the best explanation?

A

Correct Answer:
Increased blood volume in the lower extremities.
Rationale: Increased blood volume in the lower extremities is the rationale the nurse should provide for the cause of supine hypotensive syndrome during pregnancy. Hormones, fibrinogen, plasma production, and hemoglobin are not related to orthostatic hypotension.

How well did you know this?
1
Not at all
2
3
4
5
Perfectly
71
Q

A nurse is assessing a prenatal patient’s cardiovascular function. When should the nurse expect this patient’s cardiac output (CO) to begin rising?

A

Correct Answer:
Eight to 10 weeks
Rationale: Cardiac output (CO) begins to rise early in pregnancy. 8 to 10 weeks is the best answer. Twelve to 18, 20 to 24, and 34 to 38 weeks are too late.

How well did you know this?
1
Not at all
2
3
4
5
Perfectly
72
Q

A nurse is teaching a group of first-trimester prenatal patients about the discomforts of pregnancy. A patient asks the nurse, “What causes my nausea and vomiting?” The nurse knows the primary contributing factor to first-trimester emesis is:

A

Correct Answer:
Human chorionic gonadotropin.
Rationale: The primary cause of prenatal nausea and vomiting is an elevated level of human chorionic gonadotropin. Estrogen stimulates the growth of the uterus and breast tissue. Progesterone prepares the breasts for lactation and decreases uterine contractions. Prostaglandins stimulate uterine contractions.

How well did you know this?
1
Not at all
2
3
4
5
Perfectly
73
Q

A nurse is researching the topic of fluid retention during pregnancy. Which factor contributes to fluid retention?

A

Correct Answer:
Increased level of steroid sex hormones
Rationale: Increased level of steroid sex hormones contributes to fluid retention during pregnancy. Decreased serum protein influences the fluid balance. Increased intracapillary pressure and permeability influences the fluid balance. Nitrogen retention does not influence fluid balance.

How well did you know this?
1
Not at all
2
3
4
5
Perfectly
74
Q

The nurse is taking an initial history of a prenatal patient. Which sign would first indicate a positive, or diagnostic, sign of pregnancy?

A

Your Answer:
Fetal heartbeat with fetoscope at 18 weeks gestation
Rationale: If the nurse practitioner hears the fetal heartbeat with a fetoscope between 17 and 20 weeks gestation, that would indicate a positive, or diagnostic, sign of pregnancy. The examiner can detect fetal movement at 20 weeks gestation. Fetal heart movement can be seen as early as eight weeks gestation with ultrasound. Fetal heartbeat can be detected with the electronic Doppler at 10–12 weeks gestation.

How well did you know this?
1
Not at all
2
3
4
5
Perfectly
75
Q

The nurse in the prenatal clinic assesses a 26-year-old patient at 13 weeks gestation. Which presumptive (subjective) signs and symptoms of pregnancy should the nurse anticipate?

A

Correct Answer:
Excessive fatigue and urinary frequency
Rationale: Excessive fatigue and urinary frequency both are presumptive (subjective) signs and symptoms of pregnancy. Hegar’s sign, ballottement, a positive pregnancy test, Chadwick’s sign, and uterine souffle are probable (objective) signs or symptoms of pregnancy.

How well did you know this?
1
Not at all
2
3
4
5
Perfectly
76
Q

The nurse is researching the topic of uteroplacental blood flow. Which statement accurately describes funic souffle?

A

Correct Answer:
A soft blowing sound of blood that is at the same rate as the fetal heart rate.
Rationale: Funic souffle is a soft blowing sound of blood that is at the same rate as the fetal heart rate. Increased blood pulsating through the placenta, a soft blowing sound of blood that is at the same rate as the maternal pulse, and increased blood pulsating through the uterine arteries relate to uterine souffle.

How well did you know this?
1
Not at all
2
3
4
5
Perfectly
77
Q

The nurse is taking a history from a prenatal patient at seven weeks gestation. The patient states, “I don’t know if I want this baby. How will I know if I’ll be a good mother?” What is the most appropriate response by the nurse?

A

Correct Answer:
“This is a normal reaction to parenthood in the first trimester.”
Rationale: Not knowing if she wants the baby and wondering if she’ll be a good mother are normal reactions to parenthood in the first trimester. Asking a newly pregnant woman to consider an abortion or adoption is a nontherapeutic response. The patient did not introduce the topic of not wanting her baby. Not knowing if she wants the baby and wondering if she’ll be a good mother are normal reactions to parenthood and not necessarily signs of depression, and do not warrant a referral.

How well did you know this?
1
Not at all
2
3
4
5
Perfectly
78
Q

The nurse is teaching a parenting class to prospective fathers. The nurse correctly teaches that couvade refers to the:

A

Rationale: Couvade is the unintentional development of the physical symptoms of pregnancy in the father of the baby. The expectant father’s fear of hurting the unborn baby during intercourse, transition from nonparent to parent, and development of attachment and bonding behaviors are third-trimester paternal concerns.

Implementation; Health Promotion and Maintenance; Application

How well did you know this?
1
Not at all
2
3
4
5
Perfectly
79
Q

The nurse is preparing an antenatal patient for an initial assessment. What is the first task that the nurse should perform?

A

Correct Answer:
Instruct the patient to provide a clean urine specimen.
Rationale: Instructing the patient to provide a clean urine specimen is the first task that the nurse should perform in preparing an antenatal patient for an initial assessment. Providing the patient with a gown is done after obtaining the urine specimen. Preparing the patient for a pelvic exam is done after or during the physical exam. Drawing blood for routine tests is the last task performed. Lab tests may be added based on assessment data from the physical exam.

How well did you know this?
1
Not at all
2
3
4
5
Perfectly
80
Q

The nurse in the prenatal clinic is planning care for a pregnant 15-year-old patient. The nurse knows that this adolescent is at risk for which maternal complication?

A

Correct Answer:
Preeclampsia
Rationale: Adolescents are at increased risk for preeclampsia. Postpartum hemorrhage is a complication of multiparity. Hypoglycemia is a complication of diabetes. Cesarean birth is a high-risk factor for patients over 35 years of age.

How well did you know this?
1
Not at all
2
3
4
5
Perfectly
81
Q

The nurse has completed the initial assessment on four prenatal patients. Which patient is at greatest risk for a spontaneous preterm birth?

A

Correct Answer:
A 19-year-old patient with twins.
Rationale: Twins place a patient at risk for preterm labor because of the overdistention of the uterus relative to the weeks of gestation. Diabetes places the patient at risk for preeclampsia and cesarean birth. Hyperthyroid disorders are associated with an increased risk of postpartum hemorrhage. Ppatients older than 35 are at risk for preeclampsia and cesarean birth.

How well did you know this?
1
Not at all
2
3
4
5
Perfectly
82
Q

The nurse is assessing the fundal height of a patient at 12 weeks gestation. The nurse should expect the fundus to be:

A

Your Answer:
Slightly above symphysis pubis.
Rationale: The fundal height is expected to be slightly above the symphysis pubis for a patient at 12 weeks gestation. The fundus is expected to be at the level of the umbilicus at 20–22 weeks gestation, and halfway between the symphysis and umbilicus at 16 weeks gestation. It is expected to be slightly below the ensiform cartilage at 36 weeks gestation.

How well did you know this?
1
Not at all
2
3
4
5
Perfectly
83
Q

A nurse is completing an assessment on a first-trimester antepartal patient with a hemoglobin level of 10.8 g/dL. What is the priority nursing action at this time?

A

Your Answer:
Obtain an order for iron supplementation.
Rationale: Obtaining an order for iron supplementation is the priority nursing action on a first-trimester antepartal patient with a hemoglobin level of 10.8 g/dL. Referring the patient for nutritional counseling is an important intervention at 12.0 g/dL but is not the priority at this time. An order for type and cross match is not needed at this time. Transfusions may be given at levels below 6.0–8.0 g/dL. Evaluating the patient for signs of infection is the appropriate action for an elevated white blood cell count.

How well did you know this?
1
Not at all
2
3
4
5
Perfectly
84
Q

An antepartum patient tells the nurse her last period was May 18. Using Nägele’s rule, what is the correct expected date of birth?

A

Correct Answer:
February 25 (of the next year)
Rationale: February 25 (of the next year) is the correct date of birth based on the fact that the first day of her last period was May 18. Calculating with Nägele’s rule (subtract three months first, then add seven days to the first day of the last menstrual period) provides for the EDB of February 25.

85
Q

A nurse assesses four patients in the prenatal clinic. Which patient will present with the most accurate fundal height related to gestational age?

A

Correct Answer:
The patient who develops hypertension
Rationale: The patient presenting with hypertension has the most accurate fundal height related to gestation. There may be difficulty accurately palpating the fundus in a patient who develops polyhydramnios (extra amniotic fluid). The uterine size may be distorted in a patient who develops uterine fibroids. There may be difficulty accurately palpating the fundus in the patient with obesity.

86
Q

A nurse is assessing a prenatal patient at 26 weeks gestation. The nurse anticipates measuring the fundal height at __________ cm.

A

Correct Answers:
26

twenty-six
Rationale: Twenty-six cm is the anticipated fundal height in a prenatal patient at 26 weeks gestation (one cm/week between 20 and 36 weeks). A fundal height of 20 cm correlates with 20 weeks gestation, 24 cm with 24 weeks gestation, and 30 cm with 30 weeks gestation.

87
Q

A patient who is eight weeks pregnant gives the following pregnancy history to the nurse: This is her fourth pregnancy; she had one abortion at 12 weeks, she had a girl born at home at 35 weeks, and she gave birth to a stillborn at 38 weeks. Which of the following is the correct documentation for this patient’s obstetric history?

A

Your Answer:
Gravida 4 para 1111
Rationale: The TPAL approach provides more detailed information about a woman’s pregnancy history. Gravida 4 para 1111 is in her fourth pregnancy; she had one abortion at 12 weeks, she had a girl born at home at 35 weeks (who is still living), and she gave birth to a stillborn at 38 weeks. Gravida 3 para 0110 is a woman who has been pregnant three times. She had no babies born at term, had one preterm baby, had one abortion, and has no living children. She is currently pregnant. Gravida 3 para 1111 is a woman who has been pregnant three times. She had one term baby, had one preterm baby, had one abortion, and has one living child. Gravida 4 para 2102 is a woman who has been pregnant four times. She had two babies born at term (who are currently living) and one preterm baby who died. She has had no abortions. She is currently pregnant.

88
Q

A patient in the prenatal clinic complains of nausea and vomiting. Which intervention should the nurse suggest?

A

Correct Answer:
Eat dry crackers or toast before arising in the morning.
Rationale: Eating dry crackers or toast before arising in the morning is a good intervention for a patient complaining of prenatal nausea. Foods high in fiber help with constipation problems, not nausea. Brushing teeth after meals may trigger vomiting. Consuming liquids with meals may cause overdistention of the stomach.

89
Q

A second-trimester patient in the prenatal clinic complains of ankle edema. Which intervention should the nurse suggest?

A

Correct Answer:
Elevate legs when sitting.
Correct rationale: Elevating the legs when sitting will increase venous return and decrease circulatory congestion in the lower extremities. Avoiding walking may increase edema and wearing ankle socks and practicing plantar flexion will have no effect on edema.

90
Q

A prenatal patient at 10 weeks gestation is complaining of leakage of urine. Which self-care strategy should the nurse teach?

A

Correct Answer:
Wear panty liners during the day.
Rationale: The increase in leakage is typically due to a full bladder and increased pressure on the bladder from the fetus. Wearing panty liners during the day may protect clothes and emotional embarrassment from the urinary leakage that occurs with pregnancy. Adequate fluids should be maintained at 2000 mL, not decreased. The bladder should be emptied every two hours, not every hour (too frequent) or every four hours (not frequent enough).

91
Q

A prenatal patient in the third trimester of pregnancy is diagnosed with varicosities in the vulva and perineum. Which self-care strategy should the nurse teach?

A

Correct Answer:
Elevate legs level with hips while sitting or lying down.
Rationale: The hips, as well as the feet and legs, must be elevated to promote venous drainage into the trunk. Supportive hose need to be applied in the morning, rather than starting in the afternoon or evening. Changing shoes may help with back pain but not with varicosities.

92
Q

A prenatal patient in her third trimester of pregnancy complains of frequent leg cramps. She asks the nurse, “What can I do to prevent these cramps?” What is the nurse’s best response?

A

Your Answer:
Decrease milk intake to a pint a day and take calcium carbonate supplements.
Rationale: The leg cramps are often caused by an imbalance of calcium and phosphorous ratio. Decreasing milk intake to a pint a day and taking calcium carbonate are good ways to prevent leg cramps in the third trimester of a pregnancy. Increasing milk and dairy servings is not helpful and may increase the imbalance. Changing position and resting often with the feet and legs elevated helps with reduction of venous stasis, not leg cramps.

93
Q

The nurse is teaching a group of prenatal patients about nipple preparation for breast-feeding. For which patient is this procedure contraindicated?

A

Your Answer:
A patient with a history of preterm labor.
Rationale: A patient with a history of preterm labor is not an appropriate candidate for nipple preparation technique, as it might trigger uterine contractions. These techniques are appropriate for the patient with enlarged Montgomery’s tubercles, the patient with active herpetic lesions, or the patient with gestational diabetes.

94
Q

A patient in the prenatal clinic asks the nurse how she can toughen her nipples for breastfeeding. (Select all that apply.)

A

Correct Answers:
Go braless and expose nipples to the sunlight and air.

Roll the nipple.

Have your partner orally stimulate the nipples.
Correct.

Rationale: Having your partner orally stimulate the nipple is one way to toughen nipples for breast-feeding. Rolling, not rubbing, the nipples may be helpful. Going braless and exposing nipples to sun and air may be helpful; however, wearing a support bra is not.

95
Q

A nurse is teaching a group of prenatal patients about hazards in the workplace during pregnancy. The nurse correctly teaches that pregnant women who have jobs requiring long periods of standing have higher incidences of:

A

Your Answer:
Preterm birth.
Rationale: Preterm birth is an occupational hazard for women who work standing up for prolonged periods, as there is more uterine stimulation while standing than while sitting or lying down. Prolapsed cord, placenta previa, and abruptio placentae are not related to prolonged standing.

96
Q

A nurse is teaching a group of prenatal patients about the importance of exercise during pregnancy. Which patient would be the best candidate to continue with her exercise regime?

A

Correct Answer:
A patient with diagnosis of diabetes
Rationale: Exercise would be therapeutic in helping a patient with a diagnosis of diabetes control her glucose utilization. However, it would be contraindicated in a patient with an incompetent cervix and cerclage, in a patient with a diagnosis of preeclampsia, or in a patient with placenta previa due to the chance of increased uterine contractions or increasing blood pressure.

97
Q

A prenatal patient in her first trimester tells the nurse that she read that hot tubs and saunas are bad for pregnant women. The nurse explains that this is true. Why?

A

Correct Answer:
These activities increase the risk of neural tube defect and mental deficiencies in the neonate.
Rationale: The hyperthermia associated with these activities during the first trimester increases the risk of neural tube defect and mental deficiencies in the neonate. Bacteria in the water are not introduced into the cervix, because the mucous plug protects bacteria from entering. There is no relationship between hot tubs and shortness of breath in the mother, or evidence of hot tubs causing uterine contractions with preterm labor.

98
Q

A nurse working in the prenatal clinic is evaluating the weight gain pattern of four adolescents. Which adolescent is most at risk for a nutritional deficiency?

A

Correct Answer:
The adolescent who is 10 pounds underweight.
Rationale: The adolescent who is 10 pounds underweight is at the most risk. An adolescent with normal height and weight is at less risk, and overweight patients are at lower risk of developing a nutritional deficiency during pregnancy than are other patients.

99
Q

A nurse is evaluating a prenatal patient’s weight gain in the second trimester of pregnancy. The nurse knows that inadequate weight gain during pregnancy is associated with a higher risk of:

A

Correct Answer:
Giving birth to a low-birth-weight infant.
Rationale: Inadequate weight gain during pregnancy is associated with a higher risk of giving birth to a low-birth-weight infant. Gestational hypertension and placenta previa are not related to inadequate weight gain, and congenital malformations are associated with a high maternal weight.

100
Q

Which of the following statements about dietary fat is true?

A

Correct Answer:
During pregnancy, requirements for fat intake are unchanged from the prepregnancy requirements
Rationale:
- Fats have no nutritional value and can be safely eliminated from the diet (false)
- During pregnancy, fats are absorbed less completely (false)
- Fat deposits in the fetus decrease from about 12% at midpregnancy to about 2% at term (false)
- During pregnancy, requirements for fat intake are unchanged from the prepregnancy requirements (true)

101
Q

A gravida 2 para 1 prenatal patient’s ultrasound reveals twins. Her prepregnant weight is within normal limits. What is the pattern of weight gain that the nurse should recommend to the patient during the second and third trimester?

A

Correct Answer:
1½ pounds per week
Rationale: For the patient with twins, a 1½ pound-per-week weight gain is recommended. For a singleton pregnancy, the patient with a normal prepregnancy weight should gain 1 pound per week.

102
Q

A prenatal patient whose weight is within normal limits asks the nurse, “How much weight should I gain, and when?” What is the best response by the nurse?

A

Correct Answer:
“You should gain approximately 1 to 4 pounds per week during the first trimester, and then 1 pound per week during the second and third trimesters.”
Rationale:
- “You should gain approximately 1 to 4 pounds per week during the first trimester, and then 1 pound per week during the second and third trimesters.” (true)
- “During the first trimester, you should gain 0.5 to 1 pound per week, followed by 2 pounds per week during the second and third trimesters.”
- “You should gain approximately 2 to 4 pounds per week during the first trimester, and then 2 pounds per week during the second and third trimesters.”
- “During the first trimester, you should gain 1 to 3 pounds per week, followed by 0.5 pound per week during the second and third trimesters.”

103
Q

An adolescent at 18 weeks gestation complains to the nurse in the prenatal clinic about her 15-pound weight gain. What is the best response by the nurse?

A

Correct Answer:
“You should continue to gain weight, but at a slower rate.”
Rationale: “You should continue to gain weight, but at a slower rate” is the best response to an adolescent who complains to the nurse about her 15-pound weight gain. Weight gain should be around 10 pounds by 18 weeks gestation.

104
Q

A nurse is teaching a prenatal nutrition class. Which meal is highest in protein?

A

Correct Answer:
Sausage and eggs with cheese, toast, and coffee
Rationale: The sausage, eggs with cheese, toast, and coffee meal contains the highest amount of protein, at about 21 grams of protein. The toasted cheese sandwich, tomato soup, and iced tea menu contains about 12–13 grams of protein; the peanut butter sandwich, pea soup, and lemonade menu contains about 15 grams of protein; and the fortified cereal, toast, and orange juice menu contains about 12–13 grams of protein, but none of these is the highest.

105
Q

A nurse is reviewing with a prenatal patient that her lab tests reveal a low folic acid level. The nurse explains that a low folic acid level is associated with a higher incidence of:

A

Correct Answer:
Anencephaly.
Rationale: Low folic acid level is associated with a higher incidence of anencephaly or other neural tube defects. Prematurity, cardiac, and respiratory problems are not affected by folic acid.

106
Q

A prenatal patient tells the nurse about her craving for laundry starch. How should the nurse respond?

A

Your Answer:
“This craving is called pica. Try nonfat powdered milk to see if it satisfies your craving.”
Rationale: The term for craving for laundry starch is pica. Bulimia is an eating disorder characterized by bingeing and purging. Anorexia is an eating disorder characterized by extreme fear of weight gain. Vegans are pure vegetarians who will not eat any food from animal sources.

107
Q

The nurse is giving discharge instructions to a postpartum patient who is breast-feeding. The nurse should teach this patient to consume __________ calories over the pregnancy requirements.

A

Correct Answer:
200
Rationale: 200 calories over the pregnancy requirement is the appropriate amount of calories for a total of 2,500–2,700 calories per day. A 500 kcal increase from her prepregnancy, not pregnancy, requirement is another way to indicate the recommended calories.

108
Q

A nurse is teaching psychosocial development to a group of adolescents. The nurse would expect teens in which stage of adolescence to be most likely to express rebellion through experimentation with drugs, alcohol, and or sex?

A
Correct Answer:
	Middle adolescence
  	Rationale:
- Early adolescence (false)
- Late adolescence (false)
- Preadolescence (false)
- Middle adolescence (true)
109
Q

A nurse is evaluating the background of four pregnant teenagers. Which psychosocial risk is a contributing factor to teenage pregnancy?

A

Your Answer:
Sexual abuse
Rationale: Sexual abuse is a psychosocial risk factor for pregnancy in teenagers. Teens at risk for pregnancy have low self-esteem, not high, and do not participate in extracurricular school activities. The use of birth control decreases the risk for pregnancy in teens.

110
Q

Which statement made by a teenage father demonstrates his understanding of the benefits of being involved? “I will have done my part if …”

A

Correct Answer:
“I work to support the baby after it’s born.”
Rationale: Unfortunately, research indicates that there is decreasing contact with teenage moms and the babies as time passes, even if fathers are highly involved in the pregnancy and childbirth. Parental involvement during pregnancy and childbirth do make teenage moms feel less deserted, more confident in decision making, and better equipped to plan for the future.

111
Q

The nurse is writing a grant for an adolescent pregnancy prevention program. She needs to include factors that contribute to adolescent pregnancy. (Select all that apply.)

A

Correct Answers:
Lack of adult supervision

Poverty

Hispanic or African American heritage
Correct.

Rationale: Poverty, increased time spent without adult supervision, being African-American or Hispanic, low educational achievement, and a previous adolescent pregnancy are considered factors that contribute to adolescent pregnancy.

112
Q

A nurse is working with a pregnant teenager in the prenatal clinic. What would be the most important nursing action to assist this teen to meet the third-trimester developmental tasks of pregnancy? Select all that applly.

A

Correct Answers:
Reassure the patient that ambivalence is normal.

Assess the patient for discomforts of pregnancy.
Rationale: Assessing the patient for discomforts of pregnancy is a third-trimester development task. Ambivalence about the pregnancy, the need for good nutrition, and discussing continued education plans are first-trimester developmental tasks.

113
Q

Which statement made by a nurse to a 17-year-old pregnant patient at the initial prenatal visit would be most effective in developing a trusting nurse-patient relationship?

A

Your Answer:
“Since this is your first pelvic exam, I’d like to explain what will be happening.”
Rationale: Explaining unfamiliar procedures to the adolescent patient who is likely to be anxious and fearful will assist the nurse in developing a trusting relationship. Words should be weighed carefully and should be nonjudgmental and sensitive to the patient. The nurse should encourage the patient’s support system to be part of the pregnancy experience.

114
Q

The nurse assesses for complications of adolescent pregnancy in the 19-year-old patient. Which data might indicate a complication associated with adolescent pregnancy?

A

Your Answer:
Hypertension, proteinuria, edema
Rationale: Preeclampsia-eclampsia is a common complication in pregnant adolescents. Placenta previa, pregnancy-induced diabetes, and abruptio placentae are not common complications of pregnant adolescents.

115
Q

Which teaching technique would be most appropriate when teaching a sexuality class to a group of adolescents?

A

Your Answer:
Independent learning activities
Rationale: Independent learning activities allow the student to learn at their own pace and avoid embarrassment. Giggling and side conversations are expected behaviors of adolescents taking a sexuality class. They are not expected to be alert and ask questions. Graphic slides and videos may be offensive and cause the student to tune out. It is fairly typical for teens to have short attention spans.

116
Q

The nurse is planning a community program to decrease adolescent pregnancy. Based on research, what would be the most successful approach?

A

Correct Answer:
Address societal issues of poverty and education.
Rationale: Addressing societal issues of poverty and education is a successful approach to decrease adolescent pregnancy. Successful teenage pregnancy prevention programs are positive, include teens in the planning process, and begin before adolescence and continue throughout high school.

117
Q

The nurse is working with a pregnant adolescent. The patient is experiencing morning sickness, and has not been able to eat regular meals. What would be the priority nursing diagnosis?

A

Correct Answer:
Altered nutrition: less than body requirements
Rationale: Altered nutrition: less than body requirements is the correct priority nursing diagnosis for a patient who is unable to eat. Alteration in comfort, self-esteem disturbance, and ineffective individual coping may be correct secondary nursing diagnoses if more data are gathered to support them.

118
Q

A patient at 28 weeks gestation is admitted to the labor and birth unit. Which test might be used to assess the patient’s fetal status?

A
Correct Answer:
	Biophysical profile (BPP)
  	Rationale: Biophysical profile would be used to assess the patient's fetal status at 28 weeks gestation. Ultrasound for physical structure is limited to identifying the growth and development of the fetus, and does not assess for other parameters of fetal well-being. Contraction stress test is appropriate in the third trimester. Amniocentesis tests for lung maturity, not overall status.
119
Q

A prenatal patient in her second trimester is admitted to the maternity unit with painless, bright red vaginal bleeding. What test might the physician order?

A

Correct Answer:
Ultrasound
Rationale: An ultrasound for placenta location to rule out placenta previa would be ordered for a patient who presents with painless, bright red vaginal bleeding. Alpha-fetoprotein (AFP) is a test used to screen for neural tube defects. A contraction stress test is ordered in the third trimester to evaluate the respiratory function of the placenta. Amniocentesis is a procedure used for genetic diagnosis or, in later pregnancy, for lung maturity studies.

120
Q

The nurse is preparing a prenatal patient for a transvaginal ultrasound. What nursing action should be included in the preparations?

A

Correct Answer:
Place patient in lithotomy position.
Rationale: After having the patient void, assist her to a lithotomy position for a transvaginal ultrasound. Preparation for a transabdominal ultrasound includes encouraging the patient to drink 1.5 quarts of fluid, maintaining a full bladder, and applying transmission gel over the patient’s abdomen.

121
Q

A patient in her third trimester has come to the clinic for her first prenatal visit. She asks the nurse whether ultrasound can determine the baby’s age. What statement by the nurse would be the best response?

A

Your Answer:
“The estimate of gestational age may vary by one to three weeks.”
Rationale: The ability to establish fetal age accurately by ultrasound is lost in the third trimester because fetal growth is not as uniform as it is in the first two trimesters; however, ultrasound can be used to approximate gestational age within one to three weeks’ accuracy during the third trimester. A comprehensive ultrasound is used to detect anatomical defects, not gestational age. Ultrasound is not used to determine gender.

122
Q

The physician orders an ultrasound for a prenatal patient prior to an amniocentesis. The nurse explains to the patient that the purpose of the ultrasound is to:

A

Correct Answer:
Locate the placenta.
Rationale: The purpose of the ultrasound before an amniocentesis is to locate the placenta, fetus, and an adequate pocket of fluid. Determination of the gestational sac volume, measuring the crown-rump length, and measuring the biparietal diameter are aspects of assessing fetal well-being (biophysical profile, or BPP), and may or may not be done prior to the amniocentesis, depending on gestational age.

123
Q

The nurse is reviewing four prenatal charts. Which patient would be an appropriate candidate for a contraction stress test (CST)?

A

Correct Answer:
A patient with intrauterine growth retardation.
Rationale: A contraction stress test (CST) is indicated for a patient with intrauterine growth retardation (IUGR), because it will assess the respiratory function of the placenta, which may be adversely affected by the conditions causing IUGR. The contraction stress test is contraindicated for the patient with multiple gestation, an incompetent cervix, or placenta previa.

124
Q

A prenatal patient at 22 weeks gestation is scheduled for an amniocentesis. What would be an appropriate nursing action to prepare this patient for the procedure?

A

Your Answer:
Position the patient in a left lateral tilt.
Rationale: An appropriate nursing action to prepare this patient for amniocentesis would be to position the patient in a left lateral tilt to prevent supine hypotension. The skin is cleansed with povidone-iodine (Betadine), not alcohol. Rh immune globulin is appropriate only for nonsensitized Rh-negative women after the procedure. Encouraging the patient to take fluids is not appropriate prior to the procedure, because the patient may become nauseous.

125
Q

A prenatal patient at 30 weeks gestation is scheduled for a nonstress test (NST) and asks the nurse, “What is this test for?” The nurse correctly responds that the test is used to determine: (Select all that apply.)

A

Correct Answers:
Accelerations of fetal heart rate

Fetal well-being

Adequate fetal oxygenation
Correct.

Rationale: An NST documents fetal well-being by measuring fetal oxygenation and fetal heart rate accelerations, but not fetal lung maturity.

126
Q

The nurse is teaching a prenatal patient about chorionic villus sampling (CVS). The nurse correctly teaches the patient that the risks associated with CVS include: (Select all that apply.)

A

Correct Answers:
Spontaneous abortion

Rupture of membranes

Intrauterine infection
Rationale: Risks of CVS include intrauterine infection, rupture of membranes, and spontaneous abortions, as well as Rh isoimmunization and fetal limb defects.

127
Q

A prenatal patient at 30 weeks gestation is scheduled for an amniocentesis to determine fetal lung maturity. The nurse expects the lecithin/sphingomyelin (L/S) ratio to be:

A

Your Answer:
1:1
Rationale: At about 30–32 weeks gestation, the amounts of lecithin to sphingomyelin become equal, an L/S ratio of 1:1. Prior to 30 weeks gestation, the lecithin concentration is less than that of the sphingomyelin (L/S ratio of 0.5:1). After 35 weeks, the lecithin exceeds the sphingomyelin by a ratio of 2:1 or greater.

128
Q

A patient who has admitted to heavy alcohol use throughout her pregnancy just delivered a 6-pound baby. Which sign or symptom in the mother should the nurse anticipate in the 12-to-48-hour postpartum period?

A

Correct Answer:
Seizures
Rationale: As a result of alcohol dependence, the woman may have withdrawal seizures as early as 12 to 48 hours after she stops drinking. Hypertension and tachycardia are more likely to be experienced postpartum in the alcoholic mother. Fever is not expected.

129
Q

A postpartum patient who admits to heavy alcohol use asks the nurse about breast-feeding her baby. The nurse correctly teaches this patient that excessive alcohol consumption while breast-feeding may:

A

Correct Answer:
Decrease the maternal milk letdown reflex.
Rationale: Excessive alcohol consumption while breast-feeding may decrease, not increase, the maternal milk ejection reflex. Fetuses exposed to heroin in utero may experience seizure disorders as newborns. Mental retardation in the newborn may result from alcohol exposure in utero, not through consumption of breast milk.

130
Q

The nurse is teaching a patient with diabetes about insulin requirements during pregnancy. Which statement is true regarding insulin requirements?

A

Correct Answer:
Insulin needs increase late in the first trimester.
Rationale: Insulin needs increase late in the first trimester and in the third trimester. Insulin needs decrease early in the first trimester.

131
Q

A prenatal patient with diabetes asks the nurse about pregnancy-related complications for her baby from diabetes. For what is the baby at risk when the mother has diabetes? (Select all that apply.)

A

Correct Answers:
Macrosomia

Sacral agenesis

Respiratory distress syndrome
Rationale: The infant of a diabetic mother is at risk for sacral agenesis, respiratory distress, and macrosomia. Hyperactivity is not a risk factor for a newborn whose mother has diabetes.

132
Q

The nurse is caring for a laboring patient with Type I diabetes. What signs and symptoms would the nurse assess if hypoglycemia was suspected?

A

Your Answer:
Diaphoresis and disorientation
Rationale: Hypoglycemia manifests itself during labor in a Type I diabetic with diaphoresis and disorientation. There is usually hunger and decreased urination, with headache and clammy skin with blurred vision.

133
Q

A patient with Type I diabetes is admitted to the labor and birthing unit. What nursing action should the nurse perform first?

A

Correct Answer:
Assess blood sugar level.
Rationale: Assessment of the blood sugar level should be the first nursing action performed for a patient with Type I diabetes admitted in labor. Obtaining prenatal record, checking urine for protein, and obtaining a CBC would not be first actions.

134
Q

The nurse is counseling a prenatal patient regarding the need to take folic acid supplements during pregnancy. The nurse also encourages the patient to eat foods high in folic acid, such as:

A

Correct Answer:
Fresh green leafy vegetables and legumes.
Rationale: Fresh green leafy vegetables and legumes are good sources of folic acid. Fruits and fruit juice, rice and pasta, eggs, and yogurt are not sources of folic acid.

135
Q

The nurse is caring for a laboring patient with sickle cell anemia. Which therapy should the nurse anticipate the physician ordering?

A

Correct Answer:
Oxygen
Rationale: Oxygen supplementation is an anticipated therapy for patients with sickle cell anemia, to reduce the risk of their red blood cells sickling in the presence of decreased oxygen. Diuretics, magnesium sulfate, and bronchodilators are not anticipated for patients with sickle cell anemia.

136
Q

The nurse is completing a history for a new patient in the prenatal clinic. The patient states that she has had a successfully repaired ventricular septal defect with no further problems. The nurse anticipates what to order for this patient?

A
Correct Answer:
	No treatment is indicated
  	Rationale:
- Anticoagulant therapy
- No treatment is indicated (true)
- Antibiotic prophylaxis
- Cardiology evaluation with cardiac stress testing
137
Q

The nurse is providing prenatal care to an asymptomatic HIV-infected patient. Which nursing intervention should take priority?

A

Correct Answer:
Taking her temperature
Rationale: Taking her temperature to monitor for signs of fever is the nursing intervention that should take priority in an asymptomatic HIV-infected prenatal patient. The patient should have a visual examination each trimester to detect complications. Performing a hearing test and assessing reflexes are not priority interventions.

138
Q

The nurse is reviewing the lab tests of four prenatal patients. Which lab finding would support the diagnosis of hyperemesis gravidarum?

A

Your Answer:
Hypokalemia
Rationale: Potassium loss (hypokalemia), not hyperkalemia, is characteristic of hyperemesis gravidarum. Neither hypercalcemia nor hypocalcemia (low calcium) is characteristic of hyperemesis gravidarum.

139
Q

A prenatal patient is receiving home care for severe hyperemesis gravidarum. If the patient does not respond to standard treatment, the nurse will anticipate adding which of the following therapies on an outpatient basis?

A

Your Answer:
IV fluids
Rationale: Intravenous fluids may be ordered on an outpatient basis. Total parenteral nutrition would be started only if the patient is unresponsive to IV hydration. Low-fat soft diet and complex carbohydrates with limited liquids are progressive diets after the patient is stabilized for hyperemesis gravidarum.

140
Q

A patient at 30 weeks gestation is admitted to the maternity unit with vaginal bleeding. What should be the nurse’s initial action?

A

Rationale: The nurse’s initial action for a patient with vaginal bleeding at 30 weeks would be to assess blood pressure and pulse. Counting and weighing peripads, observing for pallor, clammy skin, and perspiration, and starting an intravenous infusion drip are all important actions for this patient; they are just not the initial action.

141
Q

A patient at 36 weeks gestation is admitted to the labor and birth unit. Her chief complaint is abdominal cramping with a sudden gush of clear fluid. What is the priority nursing diagnosis for this prenatal patient?

A

Correct Answer:
Risk for infection related to premature rupture of membranes.
Rationale: The priority nursing diagnosis for this prenatal patient is risk for infection related to premature rupture of membranes. Secondary diagnoses are risk for ineffective coping related to unknown outcome of pregnancy, knowledge deficit related to unfamiliarity with loss of vaginal fluids, and impaired physical mobility related to strict bed rest.

142
Q

A prenatal patient at 16 weeks gestation presents to the clinic with unexplained, bright red bleeding, cramping, and backache for the past two days. A pelvic exam reveals a closed cervix. What type of abortion does this indicate?

A

Correct Answer:
Threatened
Rationale: A threatened abortion (miscarriage) has symptoms of vaginal bleeding and backache without cervical dilation. In an imminent abortion, the internal cervical os is dilated. Although the cervix is closed in a missed abortion, other symptoms would include a regression in breast changes and a brownish vaginal discharge. Diagnosis is made based on history, pelvic exam, and a negative pregnancy test. With an incomplete abortion, the embryo has passed out of the uterus, but the placenta remains, and the internal os is slightly dilated.

143
Q

The patient asks for information about ectopic pregnancy. The nurse correctly responds by saying ectopic pregnancy is caused by: (Select all that apply.)

A

Correct Answers:
Presence of an IUD

In utero exposure to diethylstilbestrol (DES)

Pelvic inflammatory disease (PID)

Endometriosis
Correct.

Rationale: Ectopic pregnancy may be caused by PID, endometriosis, IUD implantation, or DES exposure, as well as previous history of ectopic pregnancy and previous tubal surgery.

144
Q

A patient presents to the physician’s office with complaints of right-sided abdominal pain, dizziness, and vaginal bleeding. A pelvic exam determines the patient to be 10 weeks gestation with adnexal tenderness. What diagnosis should the nurse suspect?

A

Correct Answer:
Ectopic pregnancy
Rationale: A patient with an ectopic pregnancy would present to the physician’s office with complaints of one-sided abdominal pain, dizziness, and vaginal bleeding, and will have adnexal tenderness on exam. Patients with a threatened abortion would have complaints of unexplained bleeding, cramping, or backache. A pelvic exam would reveal a closed cervix. Patients with appendicitis would have complaints of lower right-sided tenderness, low-grade fever, nausea, and often vomiting. Patients with cholelithiasis would have complaints of epigastric distress, such as fullness, distention, with vague pain in the right upper quadrant of the abdomen.

145
Q

A patient at 15 weeks gestation presents to the prenatal clinic with “prune juice”-like vaginal bleeding. Other assessment data include a hematocrit of 10 and complaints of severe nausea and vomiting. What diagnosis should the nurse suspect?

A

Correct Answer:
Hydatidiform mole
Rationale: A patient with a hydatidiform mole at 15 weeks gestation presents at the prenatal clinic with “prune juice”-like vaginal bleeding, anemia, and complaints of severe nausea and vomiting. Placenta previa symptoms include painless bright red vaginal bleeding, usually in the third trimester of pregnancy. Prolapsed cord symptoms include a trickle of bright red vaginal blood and possibly a visible cord at the vaginal opening. Abruptio placentae symptoms include vaginal bleeding (bright red or dark red), abdominal pain, and uterine tenderness.

146
Q

Your patient in her fourth month of her pregnancy is suspected to have an incompetent cervix. Which diagnostic measures might the nurse expect to be ordered to confirm the diagnosis? (Select all that apply.)

A

Correct Answers:
Determining a history of second-trimester abortions

Serial ultrasounds

Serial pelvic examinations
Correct.

Rationale: Diagnosis of incompetent cervix is determined by a history of unexplained second-trimester abortions, serial pelvic examinations, and serial ultrasound examinations, but not by determining a history of drug abuse.

147
Q

A prenatal nurse is assessing a patient at 34 weeks gestation who complains of watery vaginal discharge. What should be the nurse’s initial action?

A

Your Answer:
Test the fluid with nitrazine paper.
Rationale: Testing the fluid with nitrazine paper would be the nurse’s initial action, not preparing for a nonstress test or testing the urine for bacteria. Obtaining vaginal cultures for STIs is performed if further evaluation of the patient is required.

148
Q

A nurse is reviewing the factors important in the process of labor. Which two pelvic types are favorable for labor and vaginal delivery?

A

Correct Answer:
Gynecoid and anthropoid
Rationale: Gynecoid and anthropoid pelvis types are favorable for labor or delivery, whereas android and platypelloid pelvis types are not favorable.

149
Q

A patient at 39 weeks gestation calls the clinic nurse with complaints of pelvic pressure, diarrhea, and vaginal secretions. The nurse would correctly interpret these as signs and symptoms of a(n):

A

Correct Answer:
Impending labor.
Rationale: Pelvic pressure, diarrhea, and vaginal secretions are symptoms of impending labor. Vaginal infection may be recognized by an odor to the vaginal secretions, along with back or abdominal pain and fever. Urinary tract infection will present with a strong odor to the urine, along with pain and/or burning upon urination with possible fever. Although rupture of membranes precedes labor in 12% of cases, it likely would be accompanied by the expulsion of large amounts of amniotic fluid.

150
Q

A nurse is caring for a patient admitted to the birthing unit with rupture of membranes for two hours. A pelvic exam reveals a dilatation of 4 cm, and the presenting part is not engaged. Which possible complication should the nurse anticipate?

A

Correct Answer:
Prolapsed cord
Rationale: When a pelvic exam reveals a dilatation of 4 cm and the presenting part is not engaged, the nurse should anticipate a prolapsed cord. With placenta previa, the placenta is implanted in the lower uterine segment rather than the upper portion of the uterus, and it is not a complication of ruptured membranes or cervical dilatation. Amniotic infection is a potential complication after the membranes have been ruptured for >12 hours, especially if uterine contractions are present. Abruptio placentae is the premature separation of a normally implanted placenta from the uterine wall, and is not a complication of ruptured or cervical dilatation.

151
Q

A G4P3 patient in the transition phase of labor asks the nurse, “How much longer will it be before I have my baby?” What would be the best estimate that the nurse could provide?

A

Correct Answer:
One hour
Rationale: One hour is a reasonable estimate of time until delivery for a multipara woman in transition. Three hours or less would be a more appropriate answer for a nullipara woman.

152
Q

A laboring patient complains of nausea, vomiting, and increasing rectal pressure. She states, “I can’t take this anymore.” The nurse correctly assesses that this patient is in which phase of labor?

A

Correct Answer:
Transition
Rationale: Transition is the phase of labor where patients usually complain of nausea, vomiting, and increasing rectal pressure, and state, “I can’t take any more.” A laboring patient usually is able to cope in the latent and active phases of labor. Nausea, vomiting, and rectal pressure decrease during the second stage with the birth of the baby.

153
Q

A nurse assesses a rise in the fundal height and a sudden gush of blood from the vagina of a postpartum patient five minutes after birth. The nurse appropriately interprets these finding as:

A

Correct Answer:
Separation of the placenta.
Separation of the placenta is characterized by a rise in fundal height and sudden gush of blood five minutes after birth. Immediate postpartum hemorrhage is not characterized by a rise in fundal height. Late postpartum hemorrhage occurs 24–48 hours or more after birth. Delivery of the placenta is characterized by a decrease in fundal height.

154
Q

A nurse is caring for a patient during the fourth stage of labor. What are the expected assessment findings at this time?

A

Correct Answer:
Decreased blood pressure and increased pulse
Rationale: Decreased blood pressure and increased pulse are the expected assessment findings during the fourth stage of labor.

155
Q

A laboring patient is lying supine with a blood pressure of 88/60. What should be the initial nursing action?

A

Correct Answer:
Position patient to a lateral (side-lying) position.
Rationale: Position patient to a lateral (side-lying) position to correct the supine hypotension (88/60) due to aortocaval compression. Administration of oxygen, notifying the physician or nurse-midwife, or increasing the intravenous drip rate are not initial actions, because they will not correct aortocaval compression.

156
Q

A pregnant patient asks the nurse, “How will I know when I am close to starting labor?” The nurse correctly states that one possible sign of impending labor is:

A

Correct Answer:
Loss of weight.
Rationale: Impending labor may be indicated by a weight loss of 2.2–6.6 kg (1–3 pounds) resulting from fluid loss and electrolyte shifts produced by changes in estrogen and progesterone levels. Diarrhea, indigestion, or nausea and vomiting usually occur just prior to the onset of labor. Some women report a sudden burst of energy approximately 24–48 hours before labor. Abdominal discomfort can be a sign of false labor.

157
Q

A laboring patient complains to the nurse about intense pain located primarily in her back. Which fetal position should the nurse expect to see written on the patient’s chart?

A

Correct Answer:
Left-occiput-posterior (LOP)
Rationale: Either occiput-posterior (LOP or ROP) position of the fetus would cause a woman to complain of intense backache, as the fetal head presents a larger diameter in the posterior position. The anterior positions and transverse positions do not place additional pressure on the sacrum and are not associated with intense backache.

158
Q

The nurse is preparing to conduct a health history on a 3-year-old. When should a detailed birth history be collected by the nurse?

A

Rationale: A detailed birth history should be collected when the child’s current illness might be related to the birth history. Many variations from the norm are not related to the birth history. A review of systems provides a comprehensive overview of the child’s current state of health from a system point of view. The chief complaint is related to the child’s current problem and is not part of the history.

159
Q

The nurse is preparing to assess a child. When should the nurse begin the physical examination?

A

Correct Answer:
When the nurse first sees the child
Rationale: The physical examination begins with the nurse’s first contact with the child. The nurse observes the child’s behavior, general appearance, and skin color. All other responses occur after this point in time. The nurse will continue to observe the child while taking and recording vital signs, obtaining information on daily routines, and completing the health history interview with the parents.

160
Q

During an assessment of the neck of a 2-year-old child, the nurse notes firm, nontender, movable lymph nodes 1 cm in diameter in the cervical chain. How should the nurse explain this finding to the parents?

A

Correct Answer:
As a normal finding in a child this age
Rationale: Firm, nontender, movable lymph nodes up to 1 cm in diameter are common in young children; therefore, this is a normal finding. Minor viral URIs may not cause lymph node enlargement. Lymph node enlargement does not indicate antibiotic treatment. Diffuse lymph node enlargement can sometimes be indicative of a serious health problem, such as leukemia; however, in a child this age, cervical lymph nodes and tonsillar lymph nodes in particular are often residually enlarged.

161
Q

A 3-year-old presents to the emergency department with signs of respiratory distress. The child has epiglottitis associated with high fever, is apprehensive, and is drooling. What should the nurse avoid?

A

Your Answer:
Inspecting the child’s mouth and throat with a tongue blade.
Rationale: Insertion of a gloved finger or an instrument into the mouth (e.g., a tongue blade) might trigger complete airway obstruction. Although other aspects of the assessment can be deferred if doing them increases the child’s anxiety and hence his respiratory distress, they would not trigger complete airway obstruction. It is important to obtain weight, if possible, so that an appropriate dose of antibiotic can be administered. Lung assessment and vital signs will give more data related to the child’s condition and should be obtained if possible.

162
Q

Which statement by the mother indicates understanding of the nurse’s teaching related to a newborn?

A

Your Answer:
“I should use a blanket to swaddle my newborn when he is sleeping.”
Rationale: The infant has a larger body surface area for weight than adults. Newborns are especially susceptible to hypothermia and will need a blanket until they are old enough to maintain their body temperature. Infants breathe quite a bit faster than adults. The soft spot on the top of the infant’s head should not close until about 18 months of age. If the parents note any fullness or bulging of the fontanel, they should report it to the physician immediately, because this is an indication of neurological infant dysfunction. Yellow skin in an infant is indicative of jaundice and needs evaluation.

163
Q

A nurse is assessing a 2-year-old boy with the following vital signs: temperature 97.8°F axillary, apical pulse 100, respirations 28 breaths per minute, blood pressure 125/80 mmHg. Which action by the nurse is the most appropriate based on these vital signs?

A

Your Answer:
Reporting the blood pressure to the healthcare provider.
Rationale: All of the vital signs listed are normal for a 2-year-old except for the blood pressure. This reading is greater than the 99th percentile for a 2-year-old and should be reported promptly to the healthcare provider.

164
Q

When assessing an 18-month-old child, which technique should the nurse plan to use?

A

Rationale: The toddler generally has a high level of anxiety during physical assessment. Allowing the child to sit in the parent’s lap offers the child comfort. The child can be positioned in a variety of ways in the parent’s lap to facilitate the examination. The examining table is appropriate for other age groups. Restraining a child this age on the examining table for assessment will only increase the child’s anxiety.

165
Q

Which assessment finding in an 18-month-old would indicate the need for further evaluation?

A

Correct Answer:
Vocabulary of four words
Rationale: The 18-month-old should have a vocabulary of 7 to 20 words. The other findings are normal for an 18-month-old.

166
Q

Assessment of a 12-year-old female indicates that the teen’s BMI is in the 90th percentile. Which nursing diagnosis would be a priority for this adolescent?

A

Your Answer:
Nutrition, Altered: Risk for More than Body Requirements
Rationale: The teen’s BMI in the 90th percentile means that she is at risk for being overweight; therefore, Nutrition, Altered, Risk for More than Body Requirements would be most appropriate. Should the teen become overweight or obese, activity intolerance might become an issue. The diagnosis of altered growth and development cannot be made based on this information.

167
Q

Which findings, noted in a 2-month-old infant, require further assessment and intervention? (Select all that apply.)

A

Correct Answers:
Respiratory rate of 70

Substernal retractions
The 2-month-old has a rounded chest and uses the diaphragm as the primary muscle for breathing. Bronchovesicular breath sounds are normal in all age groups. Symptoms of respiratory distress include a respiratory rate greater than 60, adventitious breath sounds such as rhonchi, and substernal retractions. All of these require further assessment and intervention. Nursing

168
Q

A newborn is born at 38 weeks gestation weighing 2250 grams. Which is the most appropriate nursing diagnosis?

A

Your Answer:
Risk for altered body temperature
Rationale: This newborn is small for gestational age and is experiencing heat loss due to low birth weight. It is a priority of the nurse to ensure a neutral thermal environment. No information is given to indicate the other nursing diagnoses would be priorities.

169
Q

What would be considered an abnormal finding upon the initial physical assessment of the newborn?

A

Correct Answer:
A two-vessel cord
Rationale: A two-vessel clamped umbilical cord is the only abnormal physical finding listed. A two-vessel cord can indicate anomalies of the renal system. The cord should have three vessels.

170
Q

To assist the nurse in providing comprehensive care, which subjective data are pertinent to document?

A

Correct Answer:
Available support system
Rationale: Available support system is the only subjective data information provided. Height/Weight/Body Mass Index, gestational examination results, and newborn screening results all are objective assessment data.

171
Q

A frequent blood glucose test may be indicated for which newborn?

A

Correct Answer:
A newborn that is large for gestational age.
Rationale: A blood glucose evaluation should be performed on at-risk newborns, or as clinically indicated (such as for small-for-gestational-age or large-for-gestational-age infants, or if the newborn is jittery).

172
Q

efore giving a newborn the first sponge bath, the nurse must first:

A

Correct Answer:
Check the temperature.
Rationale: The nurse must make sure the newborn can maintain an adequate body temperature before exposing him to different water temperatures. Decreasing room temperature, weighing the baby, and checking capillary refill are not related to ensuring a newborn’s readiness for a bath.

173
Q

To facilitate family-newborn attachment, the nurse should do what? (Select all that apply.)

A

Correct Answers:
Assist with an interactive bath.

Encourage sibling visitation whenever possible.
Rationale: Rooming in and family visits are encouraged at all times to promote attachment. Even in the instance of IV antibiotic therapy, the newborn may still stay with the family in the hospital room.

174
Q

What symptoms would indicate respiratory distress in the newborn? (Select all that apply.)

A

Correct Answers:
Grunting

Changes in color or activity

Chest retractions

Facial grimacing
Correct.

Rationale: All are correct. The nurse must take extra care to teach parents how to recognize hallmarks of newborn respiratory distress and how to respond immediately to signs of respiratory problems. The parents learn to observe changes in color or activity, grunting or sighing sounds with breathing, rapid breathing with chest retractions, or facial grimacing.

175
Q

When signs of fatigue occur in the newborn, such as loss of eye contact, decreased muscle tension, and closure of the eyelids, the nurse immediately does what?

A

Correct Answer:
Discourages parent tactile stimulation
Rationale: Excessive handling and tactile stimulation can cause an increase in the newborn’s metabolic rate and caloric use, and can cause fatigue.

176
Q

What is the best rationale for removing the cord clamp within 24 hours?

A

Correct Answer:
It decreases the chance of tension injury to the area.
Rationale: The umbilical cord is assessed for signs of bleeding or infection. Removal of the cord clamp within 24 hours reduces the chance of tension injury to the area.

177
Q

It is important to note the newborn screen should be drawn after 24 hours from the time of delivery. What is the rationale that supports this?

A

Correct Answer:
It is well documented that there is a decrease in sensitivity of the screening if obtained before 24 hours of life, resulting in underdiagnosing of PKU.
Rationale: It is noted that there can be decreased sensitivity to testing if the screening is drawn before 24 hours of life.

178
Q

he benefits of breast milk include: (Select all that apply.)

A

Correct Answers:
Immunologic benefits

Nutritional benefits

Economic benefits

Psychosocial benefits
Correct.

Rationale: All answers are correct. Immunologic advantages of breast-feeding include varying degrees of protection from respiratory and gastrointestinal infections, otitis media, meningitis, sepsis, and allergies. Breast milk provides newborns with minerals in more appropriate doses than formula does. The concentration of iron in breast milk is much lower than that in prepared formula, it is much more readily and fully absorbed and it appears sufficient to meet the infant’s iron needs for the first six months. All components are delivered to the infant in an unchanged form. Economic benefits include cost of formula versus supplies for breast-feeding, such as pump supplies, liners, etc. It is much more expensive to utilize formula. Psychosocial advantages are associated with maternal-infant attachment. Oxytocin is released, and this hormone coincides with more mood responses and increased feelings of maternal well-being.

179
Q

A breast-feeding patient is concerned that her milk supply has begun to diminish. What is the most appropriate intervention for the patient to assist in increasing milk supply?

A

Correct Answer:
Place newborn to breast whenever possible to increase milk production.
Rationale: Putting newborn to breast to feed or suckle will stimulate milk ducts and production of milk. As the newborn is put to breast frequently, the supply will catch up with the demand if patient is well hydrated. The patient should not supplement formula; an increase in calories from fruits and/or vegetables is beneficial, but does not assist in increasing milk production; and use of a pacifier also will not assist in increasing milk production.

180
Q

Which is a true medical contraindication to breastfeeding?

A

Correct Answer:
Management of newborn galactosemia
Rationale:
- Pregnant mother (false)
- Multiple births (false)
- Fathers wants to be involved in feeding the newborn (false)
- Management of newborn galactosemia (true)

181
Q

The nurse is evaluating a new mother’s understanding of successful breastfeeding. Which of the new mother’s statements indicates a need for further instruction?

A

Correct Answer:
“I should be breastfeeding my baby a maxiumum of seven times in 24 hours.”
Rationale:
- “I should be breastfeeding my baby a maxiumum of seven times in 24 hours.” (true - this is incorrect)
- “Once I’m making enough milk, I will probably be able to hear my baby swallowing.” (false - this is correct)
- “My breasts will feel less firm after I breastfeed my baby.” (false - this is correct)
- “Five days after my baby is born, her should be producing at least six wet diapers each day.” (false - this is correct)

182
Q

A patient is postoperative day 3 from a cesarean birth. What breast-feeding position is generally the most appropriate?

A

Correct Answer:
Side-lying with a pillow behind the back and one between the legs
Rationale: The mother who has had a cesarean birth needs support so that the infant does not rest on her abdomen for long periods. When the mother is breast-feeding, she is more comfortable lying on her side with a pillow behind her back and one between her legs. The football hold is also an option, but should be a second choice for a mother recovering from a cesarean section.

183
Q

A Vietnamese patient has just delivered and has stated she would like to breast-feed. What is the nurse’s best response to the patient’s comment, “I do not want to breast-feed until my milk supply is well established”?

A

Your Answer:
“What are your thoughts and preferences about breast-feeding? I would like to help you with this process any way that I can.”
Rationale: The nurse needs to be culturally sensitive to this comment. The nurse should first ask what the patient’s cultural views are and proceed from there. In the Vietnamese culture, colostrum is not offered to the newborn and breast-feeding begins only after the milk flow is established.

184
Q

The letdown reflex may be stimulated by:

A

Correct Answer:
Maternal thoughts of the infant.
Rationale: The letdown reflex can be stimulated by the newborn’s sucking, presence, or cry, or even by maternal thoughts about the infant.

185
Q

A newborn continually falls asleep at the breast. Which intervention is appropriate when promoting effective breastfeeding by this infant?

A

Your Answer:
Removing all newborn coverings except a diaper
Rationale:
- Removing all newborn coverings except a diaper (true)
- Increasing the room temperature (false)
- Minimizing tactile stimulation (false)
- Avoiding speaking to the baby (false)

186
Q

Which is the most appropriate method for defrosting breast milk?

A

Correct Answer:
Dispense plastic container of breast milk in a glass of warm water.
Rationale: Frozen milk can be thawed by initially running cool water over the container, then gradually adding warm water until the milk is thawed. Breast milk should not be defrosted under hot running water or in boiling water. Breast milk should never be microwaved. Uneven heating patterns may alter the composition of the milk and can create hot spots that can burn the infant’s mouth.

187
Q

Nutritional status of the infant is assessed at the follow-up two-week health supervision visit. What areas always should be addressed with parents related to nutritional status of their two-week-old infant? (Select all that apply.)

A

Correct Answers:
Nutritional history from the parents

Physical exam

Developmental level of the child

Food allergies
Correct.

Rationale: All should be taken into consideration for developing a plan of care for this infant. Nutrition involves many facets, and information is gathered from many different angles to provide the very best comprehensive plan for that infant.

188
Q

The need for resuscitation of the newborn at risk can be anticipated if what risk factors are present? (Select all that apply.)

A

Correct Answers:
Sigificant intrapartal bleeding

Difficult birth

Prolonged labor
Correct.

Rationale: Neonatal risk factors for resuscitation are as follows: Nonreassuring fetal heart rate pattern; difficult birth; fetal scalp/capillary blood sample-acidosis; history of meconium in amniotic fluid; apneic episode; inadequate ventilation; male infant; prematurity; SGA; multiple births; structural lung abnormality; congenital heart disease; sepsis with cardiovascular collapse.

189
Q

The highest priority intervention the nurse must perform before resuscitating a newborn with asphyxia is:

A

Correct Answer:
Establishing effective ventilations
Rationale: Suctioning is always performed before resuscitation so that mucus, blood, or meconium is not aspirated into the lungs.

190
Q

What is the correct way to perform external cardiac massage on an infant with a detectable heart rate?

A

Correct Answer:
Place both thumbs over the lower third of the sternum with fingers wrapped around and supporting the back.
Rationale: The infant is placed properly on a firm surface. The resuscitator stands at the foot of the infant and places both thumbs over the lower third of the sternum, with the fingers wrapped around and supporting the back. The two-thumb method is preferred because it may provide better coronary perfusion pressure; however, it decreases thoracic expansion during ventilation and makes access to the umbilical cord for medication administration more difficult.

191
Q

The physiologic alterations of RDS (respiratory distress syndrome) can produce

A

Your Answer:
Hypoxia.
Rationale: RDS can cause hypoxia, respiratory acidosis, and metabolic acidosis. RDS specifically does not cause hemoglobinopathies.

192
Q

The highest-priority nursing diagnosis for a neonate experiencing RDS is:

A

Your Answer:
Impaired gas exchange related to inadequate lung surfactant.
Rationale: Altered nutrition: less than body requirements would be appropriate. Impaired gas exchange would be the highest priority, although alterations in parenting and pain could be included on the comprehensive list of nursing diagnoses.

193
Q

In neonatal resuscitation management, which of the following is not included as critical assessment data?

A

Correct Answer:
Skin color
Rationale:
- Respiratory rate (false - this is included)
- Skin color (true - this is not included)
- Heart rate (false - this is included)
- Pulse oximetry measurement (false - this is included)

194
Q

Which of the following may indicate hemolytic disease of the newborn?

A

Correct Answer:
The neonate demonstrates pleural and pericardial effusion
Rationale:
- The placenta is decreased in size (false)
- The neonate demonstrates pleural and pericardial effusion (true)
- The infant’s bilirubin level is decreased (false)
- The neonate’s spleen and liver are abnormally small (false)

195
Q

What is true of physiologic jaundice?

A

Correct Answer:
It is considered a normal process that occurs during transition from intrauterine to extrauterine life and appears after 24 hours of life.
Rationale: Jaundice stays visible until around the 10-day mark. There is a marked difference between breast-fed and bottle-fed babies and bilirubin levels; breast-fed babies tend to be higher. Physiologic jaundice appears after 24 hours of life. Pathologic jaundice usually occurs before 24 hours of life.

196
Q

Why would a physician or practitioner order a Coombs’ test?

A

Your Answer:
To determine whether jaundice is due to Rh or ABO incompatibility.
Rationale: The Coombs’ test is performed to determine whether jaundice is due to Rh or ABO incompatibility. Actual blood type of the infant does not affect Coombs’ testing, and hemoglobin and hematocrit testing will also not effect Coombs’ tests. A left shift has to do with a complete blood count; a physician or practitioner would look at a left shift if considering sepsis as a diagnosis.

197
Q

What is a common symptom of polycythemia?

A

Correct Answer:
Tachycardia, respiratory distress, hyperbilirubinemia
Rationale: The following are documented symptoms of polycythemia: Tachycardia and congestive heart failure due to the increase in blood volume; Respiratory distress with grunting, tachypnea, and cyanosis, increased oxygen need, or respiratory hemorrhage due to pulmonary venous congestion, edema, and hypoxemia; Hyperbilirubinemia due to increased numbers of red blood cells breaking down, and a decrease in peripheral pulses, discoloration of extremities, alteration in activity or neurologic depression, renal vein thrombosis with decreased urine output, hematuria, or proteinuria due to thromboembolism.

198
Q

Which intervention should the nurse suggest to the parents of a 12-month-old as the most effective way to reduce the incidence of early-childhood caries?

A

Correct Answer:
Advise the parents to provide a pacifier instead of a bottle at bedtime.
Rationale: Early-childhood caries is common in young children who take a bottle of juice or milk to bed or who breastfeed at long intervals during the night. Although brushing the child’s teeth is important, once-a-day brushing will not prevent caries in a child who sleeps with a bottle of juice or milk. Regular dentist visits generally are not recommended in a 12-month-old. Giving the child a pacifier will provide comfort and help the child sleep but will not promote tooth decay.

199
Q

A mother is concerned that her 5-month-old infant spits out his rice cereal. She thinks he dislikes it. What is the nurse’s best response to the mother?

A

Correct Answer:
“This is a normal response in some babies when they are first fed from a spoon.”
Rationale: When solid foods first are introduced, the infant might spit out the food because of the normal back-and-forth movement of the tongue. With practice, the infant will learn to feed from a spoon. Changing the type of cereal might not change the spitting out of the cereal. Infant feeders are not recommended. They can lead to choking. In addition, the child needs to learn to eat from a spoon. The infant should be ready for semisolid foods, such as infant cereal, by 4 to 6 months.

200
Q

A parent is concerned that his 4-year-old will eat only Cheerios and chicken nuggets. What is the best anticipatory guidance that the nurse can offer this parent?

A

Correct Answer:
Offer chicken nuggets and Cheerios with other foods at mealtimes.
Rationale: If chicken nuggets and Cheerios are offered with other foods at mealtimes, this will foster and establish good eating patterns. The child needs to learn to eat when food is provided. Preschool-age children should not consume more than 12 ounces of juice daily. Children should not be forced to sit at the table until they eat all of their food. This serves as a type of punishment and does not foster good eating habits.

201
Q

Which is the most appropriate anticipatory guidance to give parents relative to food allergies in infants?

A

Rationale: Introducing a new food every 3 to 5 days allows the parent time to note sensitivity and identify causative foods. Fussiness might be indicative of food intolerance but is not indicative of an allergy. A parent making his own baby food is a helpful intervention but will not prevent food allergies. Instructing parents to read all baby food labels carefully is an appropriate intervention but is not as important as allowing several days between new foods.

Nursing Process: Implementation
Category of Client Need: Physiological Integrity
Cognitive Level: Analyzing

202
Q

Nursing assessment of a 14-year-old reveals a BMI in the 90th percentile and a lifestyle that includes spending 4 hours a day playing video games and eating supper while watching television. What is the priority nursing diagnosis for this adolescent?

A

Correct Answer:
Imbalanced Nutrition: More Than Body Requirements related to excessive intake and sedentary lifestyle
Rationale: The most appropriate nursing diagnosis is the one that focuses on the core of the problem. The child is overweight because of poor eating habits and a sedentary lifestyle. Fatigue and Altered Development would be more appropriate with a child who is not receiving enough calories. Although the teen might have altered body image, there are no data given that support that. A diagnosis of Disturbed Body Image would be more appropriate with the diagnosis of anorexia nervosa.

203
Q

The parent of a 2-year-old reports that the child drinks six 8-ounce cups of milk per day and eats very little solid food. Based on this information, what should the nurse further assess for in this child?

A

Your Answer:
Iron-deficiency anemia.
Rationale: Toddlers should not drink more than 16 to 24 ounces of milk per day; drinking more keeps them from being hungry for foods and limits the amount of nutrients they receive. Milk does not contain iron; therefore, when the diet consists primarily of milk, the child is at high risk for iron-deficiency anemia. The child likely is gaining weight. There is no indication of failure to thrive. Rickets are a problem in children with severe vitamin D deficiency. Based on the information given, obesity is not a concern. Further data would be needed to suspect that the child will become obese.

204
Q

A new mother has questions about breastfeeding and infant formulas. She asks the nurse what the best kind of milk is for her full-term baby. What is the best recommendation by the nurse?

A

Correct Answer:
Breast milk for the first year
Rationale: The American Academy of Pediatrics recommends breastfeeding for the first year. In situations where breastfeeding does not occur, the child should receive iron-fortified formula for the first year. If an infant is not getting enough calories through breastfeeding, supplementing with formula would be appropriate. Human milk fortifier should only be used in premature infants less than 37 weeks’ gestation.

205
Q

The nurse provides a 2-month-old infant’s parents with information related to nutrition during the first 6 months. Which statement by the parents indicates that the teaching session was effective?

A

Correct Answer:
“We will introduce rice cereal when he is 4 months old.”
Rationale: Solid foods should be introduced at 4 to 6 months of age. The first food should be rice cereal because it has a lesser chance of allergy and is easily digested. Infant feeders are not recommended. Honey should not be given to infants because of the increased risk of botulism.

206
Q

At a 2-year-old’s checkup, measurement of weight indicates that the infant has lost 4 pounds since his last checkup. What should the nurse do first?

A

Correct Answer:
Reweigh the child.
Rationale: The nurse should reweigh the child first to make sure the weight is accurate. Two-year-olds frequently move around when being weighed, so it is most important that the initial action is to verify the results. After verifying the results, then the nurse can plot the weight on a growth chart and a full history and physical, including nutritional assessment, can be obtained.

207
Q

Nutritional assessment of a 10-year-old indicates the following findings. Which does the nurse recognize as a risk alert? (Select all that apply.)

A

Correct Answers:
Having been diagnosed with asthma

Eating one vegetable a day, generally at suppertime
Rationale: The presence of a chronic illness is a risk alert. In addition, children should eat at least three servings of vegetables a day. The child’s intake of an apple and an orange a day is appropriate. A BMI of 25% is acceptable. Just because the child does not eat broccoli or spinach does not put him at risk. There are many other vegetables to choose from. The child is participating in physical activity at least 5 days a week, and this is appropriate.

208
Q

f

A

f